Re: [obm-l] Eq. irracional

2002-07-24 Thread Bruno F. C. Leite

Não fiz as contas com cuidado, mas parece que se vc fizer

(x + 9 )^1/3 =(x - 9)^1/3 + 3,

elevar tudo ao cubo (lembre que (a+b)^3=a^3+b^3+3ab(a+b)), simplificar e 
chamar (x - 9)^1/3 de y, vc cai numa equação quadrática em y e acabou, pq 
vc acha y e depois facilmente vc obtém x.


At 00:19 25/07/02 -0300, you wrote:
>Olá amigos , to com umas dúvidas nessas dae.
>Se puderem me ajudar.
>Na primeira eu pensei na seguinte idéia para resolver , fatorar
>x + 9 até que ele ficasse do tipo (a + b )³
>para então anular com a raiz cúbica , alguém poderia me dar alguma idéia
>de como fazer ?
>
>(x + 9 )^1/3 - (x - 9)^1/3 = 3
>
>
>Essa eu não pensei em muita coisa.

Nem eu.

Bruno Leite
http://www.ime.usp.br/~brleite


>(1+(x)^1/2)^1/3 + (1-(x)^1/2)^1/3 = (5)^1/3
>
>Por favor , essas ajudas vão ser muito importantes para min !!
>
>Abraço.
>Rick.
>
>
>   
>  |-=Rick-C.R.B.=- |
>  |ICQ 124805654   |
>  |e-mail [EMAIL PROTECTED]  |
>   
>
>
>--
>Use o melhor sistema de busca da Internet
>Radar UOL - http://www.radaruol.com.br
>
>
>
>=
>Instruções para entrar na lista, sair da lista e usar a lista em
>http://www.mat.puc-rio.br/~nicolau/olimp/obm-l.html
>O administrador desta lista é <[EMAIL PROTECTED]>
>=

=
Instruções para entrar na lista, sair da lista e usar a lista em
http://www.mat.puc-rio.br/~nicolau/olimp/obm-l.html
O administrador desta lista é <[EMAIL PROTECTED]>
=



Re: [obm-l] Discussao dos problemas da IMO(para toda sao Paulo

2002-08-01 Thread Bruno F. C. Leite

Bem, apesar de ainda não ter olhado a prova da IMO, sei que logo logo vou 
ficar curioso para discutir as soluções. Eu (como estudante do IME-USP) 
gostei da idéia de se fazer isto lá no IME-USP.

Bruno Leite
http://www.ime.usp.br/~brleite


At 22:43 01/08/02 -0300, you wrote:
>ime usp seria bom
>
>
>
>Sera que voces do Rio nao pensam em nos de Sampa,
>que nao tem nem como se encontrar pra isso?Se eu
>pudesse sugeriria ao Edmilson que fizesse algo
>assim no Etapa ou no IME-Usp.,E ai o que voces
>acham?
>
>Johann Peter Gustav Lejeune Dirichlet
>  --- Rodrigo Villard Milet <[EMAIL PROTECTED]>
>escreveu: > Essa idéia de fazer em dois dias é
>boa, pois
> > cada um tem sua disponibilidade
> > de horários... eu só posso na sexta...
> > Abraços,
> >  Villard
> > -Mensagem original-
> > De: Carlos Gustavo Tamm de Araujo Moreira
> > <[EMAIL PROTECTED]>
> > Para: [EMAIL PROTECTED]
> > <[EMAIL PROTECTED]>
> > Data: Terça-feira, 30 de Julho de 2002 15:41
> > Assunto: Re: [obm-l] Discussao dos problemas da
> > IMO
> >
> >
> > >Eu tinha proposto na sexta por sugestao do
> > Marcio.O Marcelo estava no
> > >IMPA e disse que tambem preferia sexta.Eu nao
> > tenho nenhum problema na
> > >segunda,entretanto.Talvez seja bom o pessoal
> > do Rio se manifestar sobre que
> > >dia prefere.Por outro lado nao vejo problema
> > em fazer uma reuniao na sexta
> > e
> > >outra na segunda,e discutir tambem outros
> > problemas,alem dos da IMO,para
> > >aproveitar a animacao do pessoal.O que voces
> > acham ?
> > >Abracos,
> > >Gugu
> > >
> > >>
> > >>Eu posso participar se for na segunda-feira.
> > Na sexta é mais difícil.
> > >>
> > >>Luciano.
> > >>
> > >>At 15:12 29/07/02 -0300, you wrote:
> > >>>Caros colegas,
> > >>>Por sugestao do Marcio vamos fazer uma
> > reuniao informal na
> > sexta-feira
> > >>>(2/8) as 14:00 no IMPA para discutir os
> > problemas da IMO deste ano.Tragam
> > >>>suas solucoes...
> > >>>Abracos,
> > >>>Carlos Gustavo Moreira (Gugu)
> > >>>
> > >>>
> >
> >>>=
> > >>>Instruções para entrar na lista, sair da
> > lista e usar a lista em
> >
> >>>http://www.mat.puc-rio.br/~nicolau/olimp/obm-l.html
> > >>>O administrador desta lista é
> > <[EMAIL PROTECTED]>
> >
> >>>=
> > >>
> >
> >>=
> > >>Instruções para entrar na lista, sair da
> > lista e usar a lista em
> >
> >>http://www.mat.puc-rio.br/~nicolau/olimp/obm-l.html
> > >>O administrador desta lista é
> > <[EMAIL PROTECTED]>
> >
> >>=
> > >
> >
> >=
> > >Instruções para entrar na lista, sair da lista
> > e usar a lista em
> >
> >http://www.mat.puc-rio.br/~nicolau/olimp/obm-l.html
> > >O administrador desta lista é
> > <[EMAIL PROTECTED]>
> >
> >=
> > >
> >
> >
>=
> > Instruções para entrar na lista, sair da lista
> > e usar a lista em
> >
>http://www.mat.puc-rio.br/~nicolau/olimp/obm-l.html
> > O administrador desta lista é
> > <[EMAIL PROTECTED]>
> >
>=
>
>
>
>___
>Yahoo! PageBuilder
>O super editor para criação de sites: é grátis, fácil e rápido.
>http://br.geocities.yahoo.com/v/pb.html
>=
>Instruções para entrar na lista, sair da lista e usar a lista em
>http://www.mat.puc-rio.br/~nicolau/olimp/obm-l.html
>O administrador desta lista é <[EMAIL PROTECTED]>
>=

=
Instruções para entrar na lista, sair da lista e usar a lista em
http://www.mat.puc-rio.br/~nicolau/olimp/obm-l.html
O administrador desta lista é <[EMAIL PROTECTED]>
=



Re: [obm-l] ajuda importante!

2002-08-02 Thread Bruno F. C. Leite

At 21:25 02/08/02 +, you wrote:
>Olá pessoal,gostaria da ajuda de vcs nas seguintes questões:
>
>1.Sejam x,y >=0 nºs reais tais que x+y=2.Mostre q
>x^2 * y^2 *(x^2 + y^2)=<2

Escreva x=1+a, y=1-a. A sua desigualdade fica (1-a^2)^2 (a^2+1)<=1, ou seja,
(1-a^2)(1-a^2)(1+a^2)<=1, o que é óbvio pois isto é o mesmo que 
(1-a^2)(1-a^4)<=1...

Não olhei a 2, mas a 3 está abaixo.


>2.Para cada inteiro positivo k ,definamos a sequencia (a_n) por a_0=1 e
>a_n=kn+(-1)^n  * a_(n-1), pra n>=1. Determine todos os valores de k para 
>os quais 2000 é um termo da sequencia.
>
>3.Sejam x,y ,z nºs reais positivos tais que xyz=32. Determine o valor 
>mínimo de
>x^2 +4xy +4y^2 +2z^2 .

Veja que x^2+4xy+4y^2+2z^2=x^2+2xy+2xy+4y^2+z^2+z^2,
que, pela desigualdade das médias geométrica-aritmética, é >=
6 * raizsexta(16x^4y^4z^4)=6 * raizsexta(16* 32^4)=96.

Por outro lado, se (x,y,z)=(4,2,4), x^2 +4xy +4y^2 +2z^2=96, logo o mínimo 
é 96.

Bruno Leite
http://www.ime.usp.br/~brleite



>Grato!
>Adherbal
>
>
>
>
>_
>Tenha você também um MSN Hotmail, o maior webmail do mundo: 
>http://www.hotmail.com/br
>
>=
>Instruções para entrar na lista, sair da lista e usar a lista em
>http://www.mat.puc-rio.br/~nicolau/olimp/obm-l.html
>O administrador desta lista é <[EMAIL PROTECTED]>
>=

=
Instruções para entrar na lista, sair da lista e usar a lista em
http://www.mat.puc-rio.br/~nicolau/olimp/obm-l.html
O administrador desta lista é <[EMAIL PROTECTED]>
=



Re: [obm-l] CRUEL

2002-08-14 Thread Bruno F. C. Leite

At 12:21 14/08/02 -0400, you wrote:
>Num polígono convexo de n lados, quando se constrói todas as diagonais 
>aparecem  pontos de interseção entre as diagonais. Determinar o número de 
>pontos de interseção?

Vamos supor que não há duas diagonais paralelas.

Note que a cada ponto de intersecção podemos associar as duas diagonais ou 
o quadrilátero formado pelos extremos destas diagonais. Logo há uma bijeção 
entre o número de intersecções e o de quadriláteros com vértices contidos 
no conjunto de vértices do poligono...logo a resposta é binomial(n,4).

Está certo?

Bruno Leite
http://www.ime.usp.br/~brleite

=
Instruções para entrar na lista, sair da lista e usar a lista em
http://www.mat.puc-rio.br/~nicolau/olimp/obm-l.html
O administrador desta lista é <[EMAIL PROTECTED]>
=



Re: [obm-l] CRUEL

2002-08-14 Thread Bruno F. C. Leite

At 17:17 14/08/02 -0300, you wrote:
>From: "Bruno F. C. Leite" <[EMAIL PROTECTED]>
> > At 12:21 14/08/02 -0400, you wrote:
> > >Num polígono convexo de n lados, quando se constrói todas as diagonais
> > >aparecem  pontos de interseção entre as diagonais. Determinar o número de
> > >pontos de interseção?
> >
> > Vamos supor que não há duas diagonais paralelas.
> >
> > Note que a cada ponto de intersecção podemos associar as duas diagonais ou
> > o quadrilátero formado pelos extremos destas diagonais. Logo há uma
>bijeção
> > entre o número de intersecções e o de quadriláteros com vértices contidos
> > no conjunto de vértices do poligono...logo a resposta é binomial(n,4).
> >
> > Está certo?
> >
> > Bruno Leite
> > http://www.ime.usp.br/~brleite
>
>Mas e se dois quadriláteros distintos tiverem o mesmo ponto de interseção
>das suas diagonais? Ou isso nunca ocorre por que nenhuma das diagonais é
>paralelas, por hipótese sua?

Bem, isto que vc disse nao segue da hipótese que eu coloquei (veja o 
octógono regular) mas é também uma hipotese necessária para que o problema 
não dependa da forma particular do poligono
(dependa só de n)

Bruno Leite
http://www.ime.usp.br/~brleite



>=
>Instruções para entrar na lista, sair da lista e usar a lista em
>http://www.mat.puc-rio.br/~nicolau/olimp/obm-l.html
>O administrador desta lista é <[EMAIL PROTECTED]>
>=

=
Instruções para entrar na lista, sair da lista e usar a lista em
http://www.mat.puc-rio.br/~nicolau/olimp/obm-l.html
O administrador desta lista é <[EMAIL PROTECTED]>
=



Re: [obm-l] CRUEL

2002-08-15 Thread Bruno F. C. Leite

At 01:54 15/08/02 -0300, you wrote:
>From: "Bruno F. C. Leite" <[EMAIL PROTECTED]>
> > At 17:17 14/08/02 -0300, you wrote:
> > >From: "Bruno F. C. Leite" <[EMAIL PROTECTED]>
> > > > At 12:21 14/08/02 -0400, you wrote:
> > > > >Num polígono convexo de n lados, quando se constrói todas as
>diagonais
> > > > >aparecem  pontos de interseção entre as diagonais. Determinar o
>número de
> > > > >pontos de interseção?
> > > >
> > > > Vamos supor que não há duas diagonais paralelas.
> > > >
> > > > Note que a cada ponto de intersecção podemos associar as duas
>diagonais ou
> > > > o quadrilátero formado pelos extremos destas diagonais. Logo há uma
> > >bijeção
> > > > entre o número de intersecções e o de quadriláteros com vértices
>contidos
> > > > no conjunto de vértices do poligono...logo a resposta é binomial(n,4).
> > > >
> > > > Está certo?
> > > >
> > > > Bruno Leite
> > > > http://www.ime.usp.br/~brleite
> > >
> > >Mas e se dois quadriláteros distintos tiverem o mesmo ponto de interseção
> > >das suas diagonais? Ou isso nunca ocorre por que nenhuma das diagonais é
> > >paralelas, por hipótese sua?
> >
> > Bem, isto que vc disse nao segue da hipótese que eu coloquei (veja o
> > octógono regular) mas é também uma hipotese necessária para que o problema
> > não dependa da forma particular do poligono
> > (dependa só de n)
> >
>
>Bruno,
>
>eu não compreendi toda a discussão. Então vou recomeçá-la.
>
>Num polígono de n lados, cujos pares de diagonais não são paralelos, existe
>uma bijeção entre o número de interseções e o de quadriláteros com vértices
>contidos no conjunto de vértices do poligono.
>
>A minha pergunta deveria ter sido a seguinte: por que, sob essas hipóteses
>(de o polígono ser convexo e nenhum par de diagonais ser paralelo), não
>existem dois quadriláteros distintos com vértices contidos no conjunto de
>vértices do polígono cujos pontos de interseção das diagonais são o mesmo
>ponto?


desculpe, acho que não fui claro. No meu último email, eu quis dizer que 
concordo com vc, e que sua observação está certa. OU seja: "polígono 
convexo + nenhum par de diagonais paralelo" não implica  "não existem dois 
quadriláteros distintos com vértices contidos no conjunto de
vértices do polígono cujos pontos de interseção das diagonais coincidem", e 
até dei o exemplo do octógono regular (que está errado, só vi agora...pois 
tem diagonais paralelas...dã) mas a sua figura está ok.

Por isso é preciso colocar mais uma hipótese: "os pontos de encontros das 
diagonais de quadriláteros diferentes não coincidem". Sem esta hipótese, a 
resposta nao depende só de n, mas tb do formato do poligono.

Será que eu entendi o que vc pensou direito?

Bruno


>Vai em anexo uma figura.
>
>Em FIG 1: AE, BF, CG e DH são diâmetros. O problema do polígono ABCDEFGH é
>que muitas das diagonais são paralelas.
>
>Eu ACHO que trazendo A, B, C e D um pentelhésimo (em FIG 2) para dentro do
>círculo, a gente pode fazer com que todas as diagonais deixem de ser
>paralelas.
>Daí teríamos o ponto central do círculo interseção das diagonais dos
>quadriláteros ACEG e BDFH, o que invalidaria a bijeção do Bruno.
>
>Mas essa é só uma impressão minha, não sei se de fato está certo esse meu
>achismo.
>
>Eduardo.
>

=
Instruções para entrar na lista, sair da lista e usar a lista em
http://www.mat.puc-rio.br/~nicolau/olimp/obm-l.html
O administrador desta lista é <[EMAIL PROTECTED]>
=



Re: [obm-l] CRUEL

2002-08-15 Thread Bruno F. C. Leite

At 19:03 15/08/02 -0300, you wrote:
>From: "Bruno F. C. Leite" <[EMAIL PROTECTED]>
> > desculpe, acho que não fui claro. No meu último email, eu quis dizer que
> > concordo com vc, e que sua observação está certa. OU seja: "polígono
> > convexo + nenhum par de diagonais paralelo" não implica  "não existem dois
> > quadriláteros distintos com vértices contidos no conjunto de
> > vértices do polígono cujos pontos de interseção das diagonais coincidem",
>e
> > até dei o exemplo do octógono regular (que está errado, só vi agora...pois
> > tem diagonais paralelas...dã) mas a sua figura está ok.
> >
> > Por isso é preciso colocar mais uma hipótese: "os pontos de encontros das
> > diagonais de quadriláteros diferentes não coincidem". Sem esta hipótese, a
> > resposta nao depende só de n, mas tb do formato do poligono.
> >
> > Será que eu entendi o que vc pensou direito?
> >
> > Bruno
>
>Agora eu entendi o que você quis dizer, e vice-versa.
>
>E o caso onde o polígono é regular, não era essa a intenção da pergunta
>inicial? Como contar as repetições?

A pergunta inicial era só convexo! Bom, sobre essa nova pergunta sua, eu 
nao sei, vou pensar amanhã (agora são 3h...)

Bruno Leite
http://www.ime.usp.br/~brleite



>Eduardo.
>
>=
>Instruções para entrar na lista, sair da lista e usar a lista em
>http://www.mat.puc-rio.br/~nicolau/olimp/obm-l.html
>O administrador desta lista é <[EMAIL PROTECTED]>
>=

=
Instruções para entrar na lista, sair da lista e usar a lista em
http://www.mat.puc-rio.br/~nicolau/olimp/obm-l.html
O administrador desta lista é <[EMAIL PROTECTED]>
=



Re: [obm-l] Integrais pesadas!

2002-08-19 Thread Bruno F. C. Leite

At 22:24 19/08/02 -0300, you wrote:
> Você por um acaso não sabe se há alguma versão online desse 
> artigo para consulta? Procurei a 'Matematica Universitaria' nas 
> bibliotecas da USP aqui em SP mas nenhuma tinha os ultimos numeros dessa 
> revista.

tenho quase certeza que a biblioteca do ime-usp tem as "matematica 
universitaria"...vc procurou
no catalogo online em www.ime.usp.br/~bib ? Senão, a SBM vende (acho que 
por $10)

Bruno Leite
http://www.ime.usp.br/~brleite



>At 09:25 8/18/2002 -0300, you wrote:
>
>>Ha um artigo do professor Daniel Cordeiro no ultimo numero da Matematica 
>>Universitaria.
>>
>>iver wrote:
>>>Olá, será qua alguém da lista poderia mostrar um método para se calcular 
>>>inegrais como sin(x/(x+1)) e semelhantes ?? (x^x ,...)
>>>
>>>agradeço antecipadamente por qualquer resposta.
>>>por favor, se puderem me indiquem livros onde eu possa estudar sobre isso...
>
>"... a perfect formulation of a problem is already half
>its solution."
>  David Hilbert.
>-
>[]'s
>Fernando Henrique Ferraz Pereira da Rosa
>USP, IME, Estatística
>http://www.linux.ime.usp.br/~feferraz
>
>
>
>---
>Outgoing mail is certified Virus Free.
>Checked by AVG anti-virus system (http://www.grisoft.com).
>Version: 6.0.381 / Virus Database: 214 - Release Date: 8/2/2002

=
Instruções para entrar na lista, sair da lista e usar a lista em
http://www.mat.puc-rio.br/~nicolau/olimp/obm-l.html
O administrador desta lista é <[EMAIL PROTECTED]>
=



Re: [obm-l] equacoes diferenciais

2002-08-21 Thread Bruno F. C. Leite

isso é verdade, eu fiz o curso junto com o David e estava quebrando a 
cabeça com uns livros porcarias que ficam só ensinando uns truques; quando 
ele me indicou esse livro facilitou bastante, é completo e bem escrito.

Bruno Leite
http://www.ime.usp.br/~brleite



At 23:13 21/08/02 -0300, you wrote:
>Na época em q eu fiz eq. dif. ordinárias na minha graduação, eu dei uma 
>boa fuçada em todos os livros (q não estavam classificados como 
>"avançado", já q eu queria algo introdutório) da biblioteca, de modo q 
>posso te assegurar q nenhum se compara ao excelente:
>
>Differential Equations, With Applications and Historical Notes
>George Finlay Simmons (o mesmo do mais famoso "Cálculo com Geometria 
>Analítica")
>
>A exposição não poderia ser mais clara. É um livro muito didático, q 
>mistura teoremas e conversa na dose perfeita.
>Acho q os pré-requisitos são um curso semestral de cálculo e rudimentos, 
>bem rudimentares mesmo, de álgebra linear (tipo teorema de Cramer, def. de 
>base de espaço vetorial, acho q só...)
>Há aplicações p/ muito da teoria (infelizmente isso eu ainda não li), e 
>sempre uma ótima seção histórica/biográfica no final de cada capítulo.
>Realmente um dos melhores livros de matemática q eu já vi.
>
>Pena q na Amazon.com esteja "Out of Print--Limited Availability"...
>Como vejo q vc eh da USP, já aviso: tem um exemplar na biblioteca do IME, 
>de 70 e pouco, mas na Física tem dois da última (3.a) edição, de 91, que 
>está um pouco ampliada. Boa sorte!
>
>David
>
>>From: "Andre Wulff Hirano" <[EMAIL PROTECTED]>
>>Reply-To: [EMAIL PROTECTED]
>>To: <[EMAIL PROTECTED]>
>>Subject: [obm-l] equacoes diferenciais
>>Date: Wed, 21 Aug 2002 11:21:54 -0300
>>
>>Alguem pode me recomendar alguns livros sobre equaçoes diferenciais (1 e 
>>2 ordem etc..), que sejam bons é claro...
>>Valeu!
>>=
>>Instruções para entrar na lista, sair da lista e usar a lista em
>>http://www.mat.puc-rio.br/~nicolau/olimp/obm-l.html
>>O administrador desta lista é <[EMAIL PROTECTED]>
>>=
>
>
>
>
>
>=
>Instruções para entrar na lista, sair da lista e usar a lista em
>http://www.mat.puc-rio.br/~nicolau/olimp/obm-l.html
>O administrador desta lista é <[EMAIL PROTECTED]>
>=

=
Instruções para entrar na lista, sair da lista e usar a lista em
http://www.mat.puc-rio.br/~nicolau/olimp/obm-l.html
O administrador desta lista é <[EMAIL PROTECTED]>
=



Re: [obm-l] Política também é assunto da lista.

2002-08-24 Thread Bruno F. C. Leite

Caro Renner,

Acho que seu email não tem nada a ver com a lista. Não conheço este seu 
candidato (e nem quero conhecer, já escolhi o meu faz tempo), e, mesmo 
assim, votar em alguém porque um outro indicou por email é complicado, não?

Não que eu não ligue para política, mas já acho um saco ver propaganda 
política nos outdoors, na tv, nos postes, em todo lugar, e agora, no meu 
email. Eu achei seu email até meio irritante, porque tenho um certo nojo do 
PSDB.

Sou vegetariano e acho o vegetarianismo importante, mas não é por isso que 
eu vou mandar receita de bife de carne de soja para a lista, está certo?

Um abraço,

Bruno

At 00:59 25/08/02 -0300, you wrote:
>Meus amigos da lista,
>
> Acreditando que a matemática, assim como diversas outras áreas do 
> conhecimento, é diretamente relacionada à estrutura pública que o pais 
> possui, seja nos cargos executivos, legislativos ou judiciários, indico o 
> nome do Manoel Veras como deputado estadual.
> Conheço o seu trabalho e a sua vontade para melhorar a vida das 
> pessoas, principalmente no interior do Estado que ainda sofre diversos 
> problemas sociais e enfrenta uma eduação de qualidade não muito boa. 
> Apesar de esse não ser o dever direto de um Deputado, o Manoel sempre se 
> preocupou e procurou soluções para esses tipos de problemas.
> Não tenho vergonha de ser ousado e pedir, além do seu voto, o seu 
> apoio a essa candidatura, pois se trata de um Deputado que faz por merecer!
>
>
> 
>Obrigado e não esqueça: Manoel Veras - 45145

=
Instruções para entrar na lista, sair da lista e usar a lista em
http://www.mat.puc-rio.br/~nicolau/olimp/obm-l.html
O administrador desta lista é <[EMAIL PROTECTED]>
=



Re: [obm-l] Livre de Quadrados

2002-08-24 Thread Bruno F. C. Leite

Oi Rubens,

Vamos fatorar um número n livre de quadrados:

n=2^a 3^b 5^c 7^d ... (a,b,c,d...>=0)

Se tivéssemos algum dos expoentes (a,b,c,d,etc) maior que 1, um quadrado 
maior que 1 dividiria n, absurdo. Logo todos os expoentes são 0 ou 1, o que 
prova a parte (a).

Para (b), seja m^2 o maior quadrado que divide n. Então n=m^2 k. Veja que 
nenhum quadrado divide k,
logo k é livre de quadrados.

Bruno Leite
http://www.ime.usp.br/~brleite


At 22:17 24/08/02 -0300, you wrote:
>
>Colegas, tenho vários problemas que não sei se foram resolvidos certos e 
>outros que não resolvi. Se alguém quiser me ajudar... Obrigado
>
>"Um inteiro diz-se livre de quadrados se não é divisível pelo quadrado de 
>nenhum inteiro maior que 1.
>Provar que:
>i) Um inteiro é livre de quadrados se e somente se pode ser fatorado em um 
>produto de primos distintos.
>
>ii) Todo inteiro é produto de um inteiro livre de quadrados por um 
>quadrado perfeito."
>
>
>--
>Aproveite melhor a Web. Faça o download GRÁTIS do MSN Explorer : 
>http://explorer.msn.com.br/intl.asp#po

=
Instruções para entrar na lista, sair da lista e usar a lista em
http://www.mat.puc-rio.br/~nicolau/olimp/obm-l.html
O administrador desta lista é <[EMAIL PROTECTED]>
=



Re: [obm-l] n e um primo p

2002-08-24 Thread Bruno F. C. Leite

Isto segue do teorema fundamental da aritmética, que diz que todo número 
inteiro positivo pode ser fatotado em primos de uma única forma, a menos da 
ordem dos fatores.

Bruno Leite
http://www.ime.usp.br/~brleite

At 22:25 24/08/02 -0300, you wrote:
>Um probleminha:
>
>Mostrar que, dados um inteiro n e um primo  p, n pode ser escrito na forma 
>n = p^k . m, em que k>0 ou k=0 e m é um inteiro não divisível por p.
>
>Obrigado!
>
>
>--
>Aproveite melhor a Web. Faça o download GRÁTIS do MSN Explorer : 
>http://explorer.msn.com.br/intl.asp#po

=
Instruções para entrar na lista, sair da lista e usar a lista em
http://www.mat.puc-rio.br/~nicolau/olimp/obm-l.html
O administrador desta lista é <[EMAIL PROTECTED]>
=



Re: [obm-l] Relativamente Primos???

2002-08-24 Thread Bruno F. C. Leite

At 22:34 24/08/02 -0300, you wrote:
>Nunca tinha ouvido falar, mas em todo caso peço ajuda.
>
>1) Provar que 4k+3 e 5k+4 são relativamente primos, para todo inteiro k.

Se x=4k+3 e y=5k+4, veja que 5x-4y=20k+15-20k-16=-1. Se d>1 e d divide x, d 
divide y, d divide 5x-4y=-1, absurdo! Logo são primos entre si.

Esse truque é muito comum, veja por exemplo um problema da IMO59. (1a imo)

Bruno Leite
http://www.ime.usp.br/~brleite


>
>
>
>
>--
>Aproveite melhor a Web. Faça o download GRÁTIS do MSN Explorer : 
>http://explorer.msn.com.br/intl.asp#po

=
Instruções para entrar na lista, sair da lista e usar a lista em
http://www.mat.puc-rio.br/~nicolau/olimp/obm-l.html
O administrador desta lista é <[EMAIL PROTECTED]>
=



Re: [obm-l] Re: [obm-l] Re: [obm-l] Re: Área do triângulo

2002-08-24 Thread Bruno F. C. Leite

At 23:08 24/08/02 -0300, you wrote:
>Pô, coitado do Renato. Com o atraso que o gerenciador da lista tem para
>enviar os e-mails acabou tendo um monte de gente corrigindo ele.
>
>Bruno, com relação ao teorema que vc citou, ele tem algum nome especial para
>que eu posso buscá-lo em outras fontes?

Acho que não é bem um teorema, é mais um exercício. Você é da unicamp, não? 
Será que não tem
o livro "Geometry Revisited" lá? (de Greitzer e Coxeter) No IME-USP tem, 
mas não precisa do livro não, você faz sozinho. Mas se vc quiser, eu mando 
a solução para você.

Bruno Leite
http://www.ime.usp.br/~brleite



>Uma outra pergunta. Dada as medidas das medianas, é possível construir o
>triângulo com régua e compasso? Como?
>
>Obrigado
>
>Vinicius Fortuna
>
>- Original Message -
>From: "Bruno F. C. Leite" <[EMAIL PROTECTED]>
>To: <[EMAIL PROTECTED]>
>Sent: Saturday, August 24, 2002 9:12 PM
>Subject: Re: [obm-l] Re: [obm-l] Re: Área do triângulo
>
>
> > Oi,
> >
> > Posso estar falando uma besteira feia, mas quando eu estudava geometria
> > plana (há 3 anos) eu acho que tinha um teorema que dizia que dado um
> > triangulo, podemos montar um triângulo com suas medianas e a razão entre
>as
> > áreas destes triangulos é 3/4.
> >
> > Se isto for verdade, o problema fica fácil.
> >
> > Bruno Leite
> > http://www.ime.usp.br/~brleite
> >
> > At 20:04 24/08/02 -0300, you wrote:
> > >Renato,
> > >x, y e z são as medianas do triângulo e não seus lados!
> > >Um abraço!
> > >Eduardo.
> > >
> > >From: "Renato Lira" <[EMAIL PROTECTED]>
> > > > Para saber se o triangulo realmente existe, tem que obedecer as
>seguintes
> > > > regras: x + y > z ; x + z > y ; y + z > x
> > > >
> > > > Para saber sua área sabendo somente os lados: seja p o semi perimetro
> > > > (x+y+z)/2
> > > >
> > > > S = sqrt[p(p-x)(p-z)(p-y)]
> > > >
> > > >
> > > >
> > > > - Original Message -
> > > > From: "Vinicius José Fortuna" <[EMAIL PROTECTED]>
> > > > To: <[EMAIL PROTECTED]>
> > > > Sent: Saturday, August 24, 2002 7:36 PM
> > > > Subject: [obm-l] Área do triângulo
> > > >
> > > >
> > > > > Uma das questões do último campeonato de programação do site de
> > >Valladolid
> > > > > (http://acm.uva.es/problemset) era o seguinte:
> > > > >
> > > > > Dados os tamanhos x, y, z das medianas de um triângulo, calcular sua
> > >área
> > > > ou
> > > > > dizer que tal triângulo não existe.
> > > > >
> > > > > Alguém tem alguma idéia de como resolver?
> > > > >
> > > > > Obrigado
> > > > >
> > > > > Vinicius Fortuna
> > > > > IC-Unicamp
>
>
>=
>Instruções para entrar na lista, sair da lista e usar a lista em
>http://www.mat.puc-rio.br/~nicolau/olimp/obm-l.html
>O administrador desta lista é <[EMAIL PROTECTED]>
>=

=
Instruções para entrar na lista, sair da lista e usar a lista em
http://www.mat.puc-rio.br/~nicolau/olimp/obm-l.html
O administrador desta lista é <[EMAIL PROTECTED]>
=



Re: [obm-l] (sem assunto)

2002-08-24 Thread Bruno F. C. Leite

At 23:02 24/08/02 -0400, you wrote:
>Olá rapaziada...vai ai um..se alguem puder ajudar.
>1)Prove que existem infinitos primos p tais que sejam congruos a 3 modulo 4.

Acho que já madei uma solução deste problema para a lista, dê uma olhada 
nos arquivos!

>2)Qual o resto da divisão euclidiana de s=1^5+2^5+3^5+...+99^5+100^5 por 
>4?? Justifique.

Observe que você pode ignorar os números pares da soma: todos eles (2^5, 
4^5, etc) são multiplos de 4. Para os impares, observe que 
(4k+1)^5+(4k+3)^5 é sempre multiplo de 4...

Bruno Leite
http://www.ime.usp.br/~brleite


>3)Se n é um multiplo de 4, qual o resto da divisão de 1^n+2^n++8^n+9^n 
>por 10?
>Valeu
>=
>Instruções para entrar na lista, sair da lista e usar a lista em
>http://www.mat.puc-rio.br/~nicolau/olimp/obm-l.html
>O administrador desta lista é <[EMAIL PROTECTED]>
>=

=
Instruções para entrar na lista, sair da lista e usar a lista em
http://www.mat.puc-rio.br/~nicolau/olimp/obm-l.html
O administrador desta lista é <[EMAIL PROTECTED]>
=



Re: [obm-l] ajuda !!

2002-08-25 Thread Bruno F. C. Leite

At 13:21 25/08/02 +, you wrote:
>olá!
>
>>   ei, como faço pra estimar a qnt. de dígitos de ^ ?
>>(e pq q eh menor q 4* ?)
>
>--> bem, realmente eh facil ver q ^ tem menos q
>4* +1 digitos, pois 10^4 >, mas ainda fica uma aproximação ruim 
>(apesar de q com essa estimativa dê pra fzer o problema), dai tentei fzer 
><10^4/2 => ^<10^4*/2^, daí usando log2=0,301 (acho q 
>eh isso)  pode-se ter uma aproximação melhor eu acho, mas como melhorar 
>mais um pouco esta aproximação? e como saber se a aproximação q temos eh 
>suficiente pra resolver a questão??
>
>   aproveitando a deixa, como provo q se x1>=x2>=...>=xn e
>y1>=y2>=...>=yn , e zi uma permutação de yi (i=1,...,n), então
>sum(xiyi)>=sum(xizi) (i=1,...,n) ???

Oi,

Esssa é a desigualdade do rearranjo. Tem numa eureka, eu acho que 5 ou 6. 
(veja o artigo de desigualdades) Deve ter também o artigo avulso no site da 
eureka.

tem um raciocínio que ilustra essa desigualdade. Suponha que você tem 3 
caixas: uma com notas de $100, outra com notas de $10 e outra com notas de 
$1. Suponha que você pode pegar 30 de uma caixa, 20 de outra e 7 de outra, 
mas você pode escolher em qual vc pega 30, em qual vc pega 20, etc. O que 
vc faz?

Bem, é claro que você vai ordenar os dois (100>10>1) e (30>20>7) e pegar 
30*100+10*20+7*1...

Pegue a soma máxima S entre todas as possiveis somas sum(xi zi). Queremos 
provar que i=z_j. Suponha que não, temos iS, o 
que será absurdo.

Bruno Leite
http://www.ime.usp.br/~brleite



>  thanks!
>  fê!
>
>
>_
>Converse com seus amigos online, faça o download grátis do MSN Messenger: 
>http://messenger.msn.com.br
>
>=
>Instruções para entrar na lista, sair da lista e usar a lista em
>http://www.mat.puc-rio.br/~nicolau/olimp/obm-l.html
>O administrador desta lista é <[EMAIL PROTECTED]>
>=

=
Instruções para entrar na lista, sair da lista e usar a lista em
http://www.mat.puc-rio.br/~nicolau/olimp/obm-l.html
O administrador desta lista é <[EMAIL PROTECTED]>
=



[obm-l] Re:

2002-08-26 Thread Bruno F. C. Leite

At 18:37 26/08/02 -0300, you wrote:
>Será que alguém poderia me ajudar neste problema:
>
>Se p e q são inteiros positivos tais que 7/10 < p/q < 11/15 ,qual o maior 
>valor que q pode assumir?
>
>
>Obrigado.

Acho que se trocarmos "maior" por "menor", o enunciado fica mais 
interessante. Aí, saber frações de Farey (ou dar um bom chute) pode ser 
útil (mas não imprescindível)

Bruno Leite
http://www.ime.usp.br/~brleite


=
Instruções para entrar na lista, sair da lista e usar a lista em
http://www.mat.puc-rio.br/~nicolau/olimp/obm-l.html
O administrador desta lista é <[EMAIL PROTECTED]>
=



Re: [obm-l] (nenhum assunto)

2002-08-30 Thread Bruno F. C. Leite

Uma alternativa é ver (usando a mesma propriedade do mdc) que

mdc(a+b,a^2+ab+b^2) = mdc(a+b, (a+b)^2 -ab)=mdc(a+b,-ab)=mdc(a+b,ab)=d
Se p é um primo que divide d, p | ab, logo p | a ou p | b. Suponha que p | 
a. Então p não divide b (pois a e b são coprimos). Mas então p não divide 
a+b, absurdo pois d divide a+b.

Logo d=1.

Bruno Leite
http://www.ime.usp.br/~brleite


(At 04:24 31/08/02 +, you wrote:





>>1)Se a e b são números primos entre si, prove que mdc(a+b,a^2+ab+b^2)=1
>>mdc(a+b,a^2+ab+b^2) = mdc(a+b, (a+b)^2 -ab)
>>
>>Existe a propriedade que mdc(x, y) = mdc(x, y-nx)
>>fazendo x=a+b, y=(a+b)^2 - ab, n = a temos:
>>mdc(a+b, (a+b)^2 -ab) = mdc(a+b, b^2) = M
>>
>>M | b^2 => M | b
>
>--->dá procê provar isso? tem q dizer q M eh primo...aí vale! :0
>
>c ya
>Fê
>
>
>
>_
>Converse com seus amigos online, faça o download grátis do MSN Messenger: 
>http://messenger.msn.com.br
>
>=
>Instruções para entrar na lista, sair da lista e usar a lista em
>http://www.mat.puc-rio.br/~nicolau/olimp/obm-l.html
>O administrador desta lista é <[EMAIL PROTECTED]>
>=

=
Instruções para entrar na lista, sair da lista e usar a lista em
http://www.mat.puc-rio.br/~nicolau/olimp/obm-l.html
O administrador desta lista é <[EMAIL PROTECTED]>
=



Re: En: [obm-l] esclarecimento

2002-09-02 Thread Bruno F. C. Leite

Pensando assim, você pode morrer amanhã com probablilidade 1/2, pois só há 
dois eventos:
1)morrer amanhã
2)não morrer amanhã

É claro que isto está errado. O problema é que os eventos não são 
equiprováveis...

Bruno Leite
http://www.ime.usp.br/~brleite

At 14:42 02/09/02 -0300, you wrote:
>Segundo essa linha de raciocínio, a probabilidade de se obter três caras é 
>1/4. Topas jogar comigo? V aposta em três caras, e eu pago 4 por 1.
>
>JF
>
>-Mensagem Original-
>De: [EMAIL PROTECTED]
>Para: [EMAIL PROTECTED]
>Enviada em: Segunda-feira, 2 de Setembro de 2002 12:18
>Assunto: Re: [obm-l] esclarecimento
>
>A indagação surgiu a partir de uma colocação de uma colega:
>Ele disse que ao jogarmos as três moedas, o que pode ocorrer é:
>Vê duas k,k e uma c ou kkk ou cc e uma k ou ccc, por isso a probabilidade 
>1 / 4.
>É como se não importa-se a ordem de caras e coroas e sim quantas caras e 
>coroas podemos ver num lançamento.
>
>Valeu

=
Instruções para entrar na lista, sair da lista e usar a lista em
http://www.mat.puc-rio.br/~nicolau/olimp/obm-l.html
O administrador desta lista é <[EMAIL PROTECTED]>
=



Re: [obm-l] Dúvida

2002-09-06 Thread Bruno F. C. Leite

At 12:13 06/09/02 -0300, you wrote:
>Olá colegas da lista,
>
>Estou com uma dúvida e gostaria de saber se alguém poderia me ajudar:
>
>Suponha uma seqüência de números reais, crescente, tal que a diferença 
>entre termos sucessivos vai a zero.
>Será que existe um limite finito para essa seqüência?

Nem sempre. Tome por exemplo a série (a_n) onde a_n=1+1/2+1/3+...+1/n.

Bruno Leite
http://www.ime.usp.br/~brleite


>
>Muito obrigada,
>
>Carol

=
Instruções para entrar na lista, sair da lista e usar a lista em
http://www.mat.puc-rio.br/~nicolau/olimp/obm-l.html
O administrador desta lista é <[EMAIL PROTECTED]>
=



Re: [obm-l] violencia e axioma da escolha

2002-09-09 Thread Bruno F. C. Leite

At 17:16 09/09/02 -0300, you wrote:
>Não sei se entendi direito, mas, ao meu ver, não teríamos conjuntos dois a
>dois disjuntos e tal propriedade é necessária para aplicar o axioma da
>escolha (ou não?).
>
>De qualquer forma, não poderíamos mapear os bandidos nos números inteiros?
>Assim teríamos uma função de escolha que pegaria em C o bandido mapeado no
>menor inteiro tal que satisfaça aquelas condições para entrar em R. Se temos
>uma função de escolha então podemos escolhê-los independentemente do axioma
>da escolha.

Acho que o conjunto dos bandidos não precisa ser enumerável.

Bruno Leite
http://www.ime.usp.br/~brleite



>Agradeço esclarecimentos
>
>Vinicius Fortuna
>
>- Original Message -
>From: "Rogerio Fajardo" <[EMAIL PROTECTED]>
>To: <[EMAIL PROTECTED]>
>Sent: Monday, September 09, 2002 12:08 PM
>Subject: Re: [obm-l] violencia
>
>
> > Olá, Vinicius
> >
> >   Cada vez que voce "retira um elemento de C" e coloca em R, na verdade
>voce
> > mudou o conjunto C. Ou seja, cada escolha que voce fez, no processo
> > indutivo, foi sobre um conjunto diferente. É semelhante a demonstração de
> > que todo conjunto infinto possui um subconjunto enumerável, em que, dado
>um
> > conjunto V, construímos indutivamente um conjunto S colocando nele, a cada
> > passo, um elemento de V que não está em S, usando o Axioma da Escolha
> >
> >
> > >From: Vinicius José Fortuna <[EMAIL PROTECTED]>
> > >Reply-To: [EMAIL PROTECTED]
> > >To: <[EMAIL PROTECTED]>
> > >Subject: Re: [obm-l] violencia
> > >Date: Sun, 8 Sep 2002 18:41:45 -0300
> > >
> > >Oi Rogério
> > >Acho que não saquei. Em que momento foi utilizado o axioma da escolha? Eu
> > >nem tinha infinitos conjuntos! Apenas conjuntos infinitos.
> > >
> > >Até mais
> > >
> > >Vinicius
> > >
> > >- Original Message -
> > >From: "Rogerio Fajardo" <[EMAIL PROTECTED]>
> > >To: <[EMAIL PROTECTED]>
> > >Sent: Sunday, September 08, 2002 2:17 PM
> > >Subject: Re: [obm-l] violencia
> > >
> > >
> > > > É bom notar que essa solução usa o axioma da escolha (de infinitos
> > >conjuntos
> > > > não-vazios, escolhemos um elemento de cada). É essencial o axioma da
> > >escolha
> > > > para resolvê-lo?
> > > >
> > > >
> > > > >From: Vinicius José Fortuna <[EMAIL PROTECTED]>
> > > > >Reply-To: [EMAIL PROTECTED]
> > > > >To: <[EMAIL PROTECTED]>
> > > > >Subject: Re: [obm-l] violencia Date: Sat, 7 Sep 2002 23:44:58 -0300
> > > > >
> > > > >- Original Message -
> > > > >From: "Fernanda Medeiros" <[EMAIL PROTECTED]>
> > > > >To: <[EMAIL PROTECTED]>
> > > > >Sent: Saturday, September 07, 2002 8:45 PM
> > > > >Subject: [obm-l] violencia
> > > > >
> > > > >
> > > > > > Olá,
> > > > > > alguém pode dar uma ajuda nestas questões?
> > > > > > 1.a)uma "gang" tem infinitos bandidos e cada um dos meliantes tem
>um
> > > > >único
> > > > > > inimigo no interior da "gang",que ele quer matar.Prove q é
>possivel
> > > > >reunir
> > > > > > uma quantidade infinita de bandidos desta "gang", semq  haja  o
> > >risco
> > >de
> > > > >q
> > > > > > um bandido mate outro durante a reunião.
> > > > >
> > > > >Pense no seguinte algoritmo:
> > > > >Temos o conjunto C de candidatos à reunião que inicialmente contém
> > >todos
> > >os
> > > > >infinitos bandidos da gangue.
> > > > >Temos o conjunto R de bandidos selecionados para a reunião que
> > >inicialmente
> > > > >está vazio.
> > > > >
> > > > >A cada passo do algoritmo procuramos em C alguém que não que matar
> > >ninguém
> > > > >de R e ninguém em R quer matá-lo.
> > > > >Seja M o subconjunto de C de bandidos que pelo menos um de R quer
> > >matar.
> > > > >Como cada bandido de R só quer matar um, |M|<=|R|
> > > > >Então, como R é finito, M será finito e V=C-M será infinito, pois C é
> > > > >infinito.
> > > > >V será o subconjunto de C dos bandidos que ninguém de R quer matar.
> > > > >Em V procuramos um bandido que não quer matar ninguém de R, retiramos
> > >ele
> > > > >de
> > > > >C, o inserimos em R e repete-se o processo.
> > > > >
> > > > >Se sempre for possível encontrar tal bandido, o processo se repetirá
> > > > >indefinidamente e com R sempre crescendo. Assim teremos infnitos
> > >bandidos
> > > > >na
> > > > >reunião sem derramamento de sangue.
> > > > >
> > > > >Se em algum momento não for possível encontrar um bandido em V, é
> > >porque
> > > > >todos os bandidos de V querem matar alguém de R. Ou seja, ninguém de
>V
> > >quer
> > > > >matar outro de V. Pegamos, então, V como o conjunto de bandidos para
>a
> > > > >reunião. Como V é infinito, teremos infinitos participantes na
>reunião.
> > > > >
> > > > > > b)Se cada bandido tiver um nº finito mas indefinido de inimigos(um
> > > > >bandido
> > > > > > pode ter 2 inimigos, outro somente 1, um terceiro pode ter 20 e
> > >assim
> > > > >por
> > > > > > diante).Será sempre possivel promover uma reunião com infinitos
> > >bandidos
> > > > >sem
> > > > > > risco de derramamento de sangue?
> > > > >Não é possível. Existe um contra-exemplo:
> > > > >Ordene os bandidos formando u

Re: Problema

2001-02-25 Thread Bruno F. C. Leite


-Mensagem original-
De: Rodrigo Villard Milet <[EMAIL PROTECTED]>
Para: Obm <[EMAIL PROTECTED]>
Data: Domingo, 25 de Fevereiro de 2001 21:42
Assunto: Problema


Como eu faço pra provar que qualquer natural maior que 11 pode ser escrito
como a soma de dois números compostos ??
¡ Villard !



Uma solução não muito bonita é a seguinte:

1) Isso é óbvio se o número (vou chamar de n) é par.
2) Isso também é fácil de verificar se n<40. (mas eu não verifiquei...)
3) Se n é ímpar e maior que 40, então n=10m+k, onde k=1, 3, 5, 7 ou 9. (e
m>=4)
Se k=9 acabou (9 é composto e 10m é composto).
Se k=1,5 ou 7 escreva n como 10(m-2) + (20+k), e 20+k=21,25 ou 27(todos
compostos), logo acabou de novo.
Se k=3 escreva n como 10(m-3) + (30+k) ou seja 10(m-3) + 33, e acabou
também.

Veja que m-2>m-3>0, logo nenhuma das parcelas é nula.

Bruno Leite




Problema com compasso

2001-02-26 Thread Bruno F. C. Leite

Dado o segmento AB, ache o ponto médio de AB, USANDO SOMENTE O COMPASSO.

Bruno Leite




Re: somatorio

2001-03-12 Thread Bruno F. C. Leite

Tem uma fórmula que aproxima MUITO bem esse somatório: seja
S(n)=1+1/2+1/3+...1/n.

Então S(n)= ln(n) + gama + 1/(2n) - 1/(12n^2) + 1/(240n^4) onde gama é a
constante de Euler 0.57721566490153286060651209008240243104215933593992

Bruno Leite




Re: Problema

2001-03-15 Thread Bruno F. C. Leite


-Mensagem original-
De: Marcelo - EPD <[EMAIL PROTECTED]>
Para: [EMAIL PROTECTED] <[EMAIL PROTECTED]>
Data: Quinta-feira, 15 de Março de 2001 16:13
Assunto: Problema


Gostaria de um modelo matemático para a resolução do seguinte problema:

   "Uma pessoa comprou uisque à R$ 70,00 a garrafa e vodka à R$ 50,00 a
garrafa. Gastou R$ 530,00.  Quantas garrafas de vódka e quantas de uísque
essa pessoa comprou?"

Bom, temos 70x + 50y =530, onde x=numero de garrafas de uisque e y= numero
de garrafas de vodka.

Entao 7x+5y=53. Como x e y devem ser inteiros positivos, o mais facil eh ir
testando solucoes (faça x=0,1... e veja se y é inteiro) Voce vai achar x=4 e
y=5. Para ver que é a unica solucao, 53=7x+5y>=7x logo 0<=x<=53/7.
Analogamente voce pode achar restricoes para y.

Bruno Leite








Re: Livros

2001-03-16 Thread Bruno F. C. Leite

Olha, o livro pelo qual eu estudei isso foi "An introduction to the Theory
of Numbers" de Ivan Niven. Também é bom o livro de Hardy e Wright. Esses
livros vc acha na livraria cultura ou na amazon, por exemplo.

Se vc quiser um mais baratinho tem um livro de Álgebra de dois autores:
Polcino e S.Coelho. Lá tem essas coisas que vc quer, só que com mais cara de
álgebra (tem os teoremas de congruências: fermat, euler, wilson) Não é tão
completo, do ponto de vista de teoria dos números (não tem congruências
quadráticas, etc)

E, por último, se vc não faz tanta questão de ser um LIVRO, vc pode pegar o
artigo do Gugu em alguma Eureka (acho que 2) que é exatamente sobre
divisibilidade e congruências (e aliás é de graça...)

Bruno Leite


-Mensagem original-
De: Igor Castro <[EMAIL PROTECTED]>
Para: [EMAIL PROTECTED] <[EMAIL PROTECTED]>
Data: Sexta-feira, 16 de Março de 2001 23:53
Assunto: Livros


Alguém sabe algum livro que tenha a teoria de congruencia, divisiblidade e
etc, e que seja bom para me indicar?





=?x-user-defined?q?Re:_Profissional_da_Matem=E1tica?=

2001-03-23 Thread Bruno F. C. Leite

Bom, o Rogério é meu colega do IME-USP, o que ele falou está certo, mas tem
umas coisas ruins que ele não falou:

1-Os cursos de física que tivemos foram um lixo. Juro que você vai aprender
a odiar Física por pelo menos uns 5 anos.

2-O Laboratório de Física é a maior perda de tempo que você vai ter na sua
vida. Por exemplo, vc vai ter que determinar experimentalmente o valor da
aceleração da gravidade com um cronômetro e uma régua...é lógico que isso
não pode dar em coisa boa. Um colega nosso achou g=30 m/s^2 , depois de 3
horas de medições. E isso não é nada.

3-Um monte de cursos interessantes são cheios de pré-requisitos, o que,
traduzindo, significa que vc não pode fazer cursos mais avançados pq eles
nem deixam vc se matricular no curso.

Bom, eu não vou ser tão resmungão: a biblioteca é bem completa, vc tem
computadores para fazer seus trabalhos, etc. Eu acho que vc devia visitar o
instituto e falar com algum professor, o que aliás foi o que eu fiz em 1999,
quando eu estava indeciso. Sobre o negócio do bacharelado e licenciatura
você pode se informar na página www.ime.usp.br ou mandar um email para algum
responsável da comissão de graduação.

Nessa página tem todos os cursos que temos no bacharelado em matemática.

Bruno Leite



-Mensagem original-
De: Rogerio Fajardo <[EMAIL PROTECTED]>
Para: [EMAIL PROTECTED] <[EMAIL PROTECTED]>
Data: Sexta-feira, 23 de Março de 2001 10:36
Assunto: Re: Profissional da Matemática


>Olá, futuro colega
>
>   Estudo Matemática na USP e não me arrependo da escolha. Na USP, o
>primeiro ano de Matemática é o ciclo básico, a partir do segundo vc escolhe
>entre: estatística, matemática e matemática aplicada. A primeira opção tem
>um mercado de trabalho muito bom e é mais prático. O curso de Matemática
>(que é o que eu estudo) é mais voltado para formação de pesquisadores. É
>essencial, para quem faz matemática pura, que continue com o mestrado e
>doutorado.
>Durante o bacharelado vc pode fazer iniciação científica: vc escolhe um
>assunto q te interessa e procura uma professor q está pedindo aluno de
>iniciação científica. Ele direcionará o q vc deve estudar e passará alguns
>problemas pra vc pensar. Vc pode pedir uma bolsa para a CNPq ou FAPESP e
>será pago pra estudar. Creio que é a melhor coisa do bacharelado em
>matemática da USP: a iniciação científica.
>   O curso de matemática aplicada é voltado para o mercado de trabalho,
como
>vc perguntou. No final do curso, vc escolhe um assunto à que vc quer
aplicar
>a matemática. Vc pode escolher qq coisa, desde que sua opção seja aprovada.
>Então vc fará um bloco de disciplinas da área q vc escolheu: Engenharia,
>computação, economia, biologia, etc (os mais escolhidos são economia e
>computação) e atuará ajudando esses profissionais em suas "continhas".
>   As áreas de economia e computação pedem bastante profissionais de
>matemática, mesmo se vc fizer matemática pura, embora essa seja mais
voltada
>à formação acadêmica.
>
>Rogério
>
>
>>From: [EMAIL PROTECTED]
>>Reply-To: [EMAIL PROTECTED]
>>To: Lista de Discussao <[EMAIL PROTECTED]>
>>Subject: Profissional da Matemática
>>Date: Wed, 21 Mar 2001 22:07:28 -0300
>>
>>Sou estudante que concluiu o ensino médio recentemente,moro em Sao Paulo
>>e gostaria de obter algumas informações sobre o curso de Bach. em
>>Matemática.
>>Queria saber onde esse profissional atua, se eh possivel ele se associar
>>com engenheiros, biologos, geologos, etc, e que opções de cursos de pós
>>graduação são recomendáveis atualmente para o graduado.
>>Outra dúvida: eh possível complementear o curso de bach. com o de
>>licenciatura?
>>Quais os beneficios?
>>Agradeceria também se pudessem me informar sobre o curso de Matemática na
>>USP.
>>
>>Obrigado e abraços,
>>Flavio Daher.
>>
>>
>>
>>
>>___
>>
>>http://www.zipmail.com.br O e-mail que vai aonde você está.
>>
>>
>>
>
>_
>Get Your Private, Free E-mail from MSN Hotmail at http://www.hotmail.com.
>




Re: ajuda

2001-04-18 Thread Bruno F. C. Leite

Estou me lembrando de um problema muito legal: uma pessoa escolhe um número
de 0 a 15, a outra pessoa tem que descobrir que número é, fazendo perguntas
com resposta "sim" ou "não". O detalhe é que o cara que pensou no número
pode mentir 1 vez se quiser. Qual é o número mínimo de perguntas que são
suficientes para descobrir o numero pensado?

Bruno


-Mensagem original-
De: Alek <[EMAIL PROTECTED]>
Para: [EMAIL PROTECTED] <[EMAIL PROTECTED]>
Data: Domingo, 15 de Abril de 2001 08:05
Assunto: Re: ajuda


>Eu responderia que o menor numero de perguntas é sete.
>
>Como cheguei a este numero?
>A primeira coisa foi lembrar de uma aula de digital onde estava aprendendo
>umas das formas de um circuito quantizar um valor, ou seja, passa-lo para
>binario, e este era o metodo que gastava menos instruçoes, era mais ou
>menos assim.
>
>Pergunta se o numero é menor que o numero mediano do universo em que se
>esta trabalhando no momento
>
>No caso sao 100 numeros
>
>Umas sequencias possiveis seriam
>
>50s; 25s; 13s; 7n; 10s; 8n; 9s -> 8
>50s; 25s; 13s; 7n; 10s; 8n; 9n -> 9
>
>25s = é menor que 25? Sim
>7n = é menor que 7? Nao
>
>
>Alek
>
>At 21:01 14/04/01 -0400, you wrote:
>> Pensei num número inteiro no intervalo de 1 até 100 e você deve
>> descobrir
>>qualé. Para ajudar, responderei, apenas com sim ou não, a
>>qualquer pergunta.
>> Qual é o menor número de perguntas que permite descobrir o número?
>




Re: ajuda

2001-04-18 Thread Bruno F. C. Leite

Explicando melhor:

A pessoa A pensa num número de 0 a 15. A pessoa B tem que adivinhar o número
que A pensou fazendo perguntas cuja resposta seja "sim" ou "não", por
exemplo: "o número é maior que 4?", etc.

a)Mostre que com no máximo 4 perguntas, B consegue acertar o número.

b)Se A puder mentir uma vez no máximo (ele pode mentir se quiser, não é
obrigado), quantas perguntas são necessárias?

Acho que agora está certinho.

Bruno
-----Mensagem original-
De: Bruno F. C. Leite <[EMAIL PROTECTED]>
Para: [EMAIL PROTECTED] <[EMAIL PROTECTED]>
Data: Quarta-feira, 18 de Abril de 2001 22:42
Assunto: Re: ajuda


>Estou me lembrando de um problema muito legal: uma pessoa escolhe um número
>de 0 a 15, a outra pessoa tem que descobrir que número é, fazendo perguntas
>com resposta "sim" ou "não". O detalhe é que o cara que pensou no número
>pode mentir 1 vez se quiser. Qual é o número mínimo de perguntas que são
>suficientes para descobrir o numero pensado?
>
>Bruno
>
>
>-Mensagem original-
>De: Alek <[EMAIL PROTECTED]>
>Para: [EMAIL PROTECTED] <[EMAIL PROTECTED]>
>Data: Domingo, 15 de Abril de 2001 08:05
>Assunto: Re: ajuda
>
>
>>Eu responderia que o menor numero de perguntas é sete.
>>
>>Como cheguei a este numero?
>>A primeira coisa foi lembrar de uma aula de digital onde estava aprendendo
>>umas das formas de um circuito quantizar um valor, ou seja, passa-lo para
>>binario, e este era o metodo que gastava menos instruçoes, era mais ou
>>menos assim.
>>
>>Pergunta se o numero é menor que o numero mediano do universo em que se
>>esta trabalhando no momento
>>
>>No caso sao 100 numeros
>>
>>Umas sequencias possiveis seriam
>>
>>50s; 25s; 13s; 7n; 10s; 8n; 9s -> 8
>>50s; 25s; 13s; 7n; 10s; 8n; 9n -> 9
>>
>>25s = é menor que 25? Sim
>>7n = é menor que 7? Nao
>>
>>
>>Alek
>>
>>At 21:01 14/04/01 -0400, you wrote:
>>> Pensei num número inteiro no intervalo de 1 até 100 e você deve
>>> descobrir
>>>qualé. Para ajudar, responderei, apenas com sim ou não, a
>>>qualquer pergunta.
>>> Qual é o menor número de perguntas que permite descobrir o número?
>>
>




Re: Frações

2001-04-23 Thread Bruno F. C. Leite

Acho que ele quis dizer que a/b < (a+c)/(b+d) < c/d, se a/b
Para: [EMAIL PROTECTED] <[EMAIL PROTECTED]>
Data: Segunda-feira, 23 de Abril de 2001 08:35
Assunto: Re: Frações


>Olá Rodrigo,
>Acredito que não seja verdadeiro a proposição seguinte:
>a/b < c/d [Image]a/b < (a+b)/(c+d) < c/d
>Verifique isto no exemplo seguinte:
>2/3 < 4/5 , entretanto   é falso que  2/3< 5/9< 4/5, pois, 2/3 > 5/9  (
>verifique !!!)
>PONCE
>
>Rodrigo Villard Milet wrote:
>
>> Note que dado a/b < c/d, temos a/b < (a+b)/(c+d) < c/d ( Verifique
>> !)Daí, temos r = 45 + 59 = 104 e s = 80 + 61 = 141 < 200.Suponha q
>> existe outro par r,s, ou seja, suponha que existam r` e s`, tais que
>> 45/61>r`/s`>59/80. Daí, existem duas possibilidades : r'/s' entre
>> 59/80 e 104/200 ou  entre 45/61 e 104/200 após fazer algumas
>> contas, vc chega a um absurdo ! Se ninguém mandar a solução, eu
>> escrevo...Abraços, ¡Villard!-Mensagem original-
>> De: Fábio Arruda de Lima <[EMAIL PROTECTED]>
>> Para: [EMAIL PROTECTED] <[EMAIL PROTECTED]>
>> Data: Quarta-feira, 18 de Abril de 2001 23:47
>> Assunto: Frações
>>
>>
>>  Olá amigos,(Olimpíada Britânica/87)Ache o par de inteiros r
>>  e s, tal que 0r/s>59/80 Além disso, prove
>>  que existe apenas um único par r e s.Um abraço.Fábio
>>
>




Re: ITA

2001-05-04 Thread Bruno F. C. Leite

Eu lembro que no meio do 3º colegial eu comecei a me preocupar só com
Matemática, comecei a ver problemas de olimpíadas, Cálculo, Álgebra Linear,
li um livro de Teoria dos Números, etc. Assim, acabei estudando pouco outras
matérias, e, por exemplo, li somente 5 dos 10 livros que a Fuvest pediu.
Para mim isso foi bom, porque no fim das contas eu passei na Fuvest e ainda
entrei na faculdade sabendo já um pouco de cálculo, etc. Mas no 1º  e 2º
colegial eu tinha estudado as outras matérias, então deu para eu me virar
bem na prova da primeira fase.

Mas a Fuvest é bem mais fácil que o ITA! Eu passei no ITA mas eu **quebrei a
cara** na prova de física. Se vc não estudar direito ou não fizer um
cursinho especializado em ITA vc vai boiar como eu - tinha enunciado que eu
nem ao menos sabia do que eles estavam falando. Eu conversei com o menino
que foi o 1º colocado na olimpíada brasileira de Física (hoje ele está no
MIT) e ele disse que chutou 1/4 da prova! A prova de matemática tem questões
fáceis mas MUITO trabalhosas e o tempo é o que mais atrapalha, com toda
certeza.

Bruno


-Mensagem original-
De: Fábio Arruda de Lima <[EMAIL PROTECTED]>
Para: [EMAIL PROTECTED] <[EMAIL PROTECTED]>
Data: Sexta-feira, 4 de Maio de 2001 22:04
Assunto: ITA


Caro Gustavo,
Se realmente é isto que você quer, dar-te-ei algumas dicas. Inicialmente,
entre numa turma preparatória IME/ITA. Esta não é a solução, porém vai te
ajudar muito.
Dedique-se as matérias do concurso, estudando todo o programa de matérias.
Não deixe de estudar nada, pois uma questão fácil de um assunto não
estudado, torna-se muito difícil. Se forem difíceis as questões do assunto
que você estudou, teremos um grande problema. Não abandone matérias como
português e línguas (deixei de ser 1º colocado no IME por causa de
português - fui 2º). Tente estudar o maior número de horas possível. Resolva
todas as provas anteriores, elas dão uma boa preparação. Faça um estudo
sólido e consistente. Decorar física, química ou matemática não é um bom
negócio. Tente entender os conceitos e deduzir por si mesmo as fórmulas. E
tenha sempre uma coisa em mente, se o Fábio Arruda passou então eu passo
(consegui muitas coisas pensando assim - se alguém passou, então eu posso
passar). Nada na vida é impossível. Tudo é uma questão de escolha e
sacrifícios. Se você sacrificar outros afazeres, certamente você terá tempo
suficiente.
Espero ter ajudado. Um abraço
Fábio Arruda





  - Original Message -
  From: Gustavo Martins
  To: [EMAIL PROTECTED]
  Sent: Friday, May 04, 2001 1:13 PM
  Subject: Re: Aprendendo mat. sem perder o resto


  Eu quero é Eng. Aer. no ITA mesmo. Se for realmente impossível isso, eu
faço física.

  []s
  Gustavo
- Original Message -
From: Fábio Arruda de Lima
To: [EMAIL PROTECTED]
Sent: Thursday, May 03, 2001 1:11 PM
Subject: Re: Aprendendo mat. sem perder o resto


Bom Gustavo,
Tudo depende do seu objetivo. Quando eu terminei o 2º grau, meu desejo
era fazer o IME, fui parar na Turma IME/ITA, abandonei as matérias tipo
Biologia, História,...
Entretanto, se este não for o seu objetivo, não deixe de estudá-las.
Principalmente, línguas e português. Alguns colegas do meu tempo de IME
estão na Microsoft (desenvolvendo coisas for Windows), porém eles
continuaram estudando inglês e espanhol. Pense bem, esta é uma decisão
difícil.
Um abraço
Fábio Arruda
  - Original Message -
  From: Gustavo Martins
  To: [EMAIL PROTECTED]
  Sent: Thursday, May 03, 2001 4:43 PM
  Subject: Aprendendo mat. sem perder o resto


  Colegas:

  Estou no 3º ano do Ens. Médio e percebi que se eu desejo aprender
*bem* a Matemática e outras matérias exatas, tenho que ter dedicação quase
exclusiva, ficando com pouquíssimo tempo para estudar os "outros" assuntos
(biologia, geografia, etc). Porém, se eu tiver que fazer isso, posso me dar
mal. Creio que alguns já passaram por esse problema e podem me dar algum
tipo de sugestão para que eu possa aprender bem as matérias exatas e sobrar
algum tempo para as outras. Qualquer ajuda serve.

  Atenciosamente,
  Gustavo





Re: Funções... (Iezzi)

2001-11-25 Thread Bruno F. C. Leite

Normalmente se convenciona que uma somatória vazia tem valor zero, assim 
como um produto vazio tem valor 1. para n=0 a sua soma é vazia, e, com essa 
convenção, a sua fórmula vale sempre.

Aliás, não acho que uma fórmula fechada não possa ter algo do tipo

f(n)=... se n=...
e
f(n)= caso contrário

Bruno Leite

PS A sua fórmula ainda pode ser mais simplificada!!

At 16:11 25/11/01 -0200, you wrote:
> Olá..
>
> Estou aqui resolvendo um problema de funções do Iezzi, mas como 
> para esse tipo de exercício 'dissertativo' não há resposta nas últimas 
> páginas, não sei se cheguei a solução correta.
>
>"Seja f uma função, definida no conjunto dos números naturais, tal que:
> f(n + 1) = 2f(n) + 3
>para todo n natural.
>a) Supondo f(0) = 0, calcule f(1),f(2),f(3),f(4),... e descubra a "fórmula 
>geral" de f(n).
>b) Prove por indução finita a fórmula descoberta."
>(in IEZZI, Gelson FME vol 1. pp 157)
>
>Fazendo f(1), f(2), f(3) etc.. achamos:
>f(1) = 3, f(2) = 9, f(3) = 21,f(4) = 45, f(5) = 93 ... f(n) = ?
>"expandindo" as contas, temos:
>f(1) = (0.2) + 3
>f(2) = (((0.2) + 3).2) + 3
>f(3) = 0.2) + 3).2) + 3).2) + 3
>f(4) = (0.2) + 3).2) + 3).2) + 3).2)+3
>f(5) = ((0.2) + 3).2) + 3).2) + 3).2)+3).2 + 3
>
>Tomando n = 3 e desenvolvendo:
>f(3) = 3.2.2 + 3.2 + 3
>o mesmo para n = 4:
>f(4) = 3.2.2.2 + 3.2.2 + 3.2 + 3
>ou 3.2³ + 3.2² + 3.2 + 3
>Isso decorre de que n+1 é dado por n.2 + 3..
>Colocando o 3 em evidência.. e notando que a maior potência de 2 é igual a 
>n-1:
>f(n) = 3(2^(n-1) + 2^(n-2) +   + 2^1 + 2^0)
>ou ainda f(n) = 3. somatória[para k = 0 até n - 1] 2^k
>A fórmula funciona para qualquer n pertencente aos naturais e diferente de 
>zero.
>Daí que vem minha dúvida... a fórmula que eu achei pode ser considerada 
>'termo' geral, se não é válida para 0?
> Alguém tem alguma idéia de outra fórmula geral?
>
>
>
>Grato pela atenção..
>
>
>
>
>"Against stupidity, the Gods themselves contend in vain",
> Friedrich von Schiller's
>-
>[]'s
>{O-Grande-Mentecapto}
>[EMAIL PROTECTED]
>
>




Re: pascal

2001-12-01 Thread Bruno F. C. Leite

O cara que está na linha n e na coluna k (existem linha 0 e coluna 0)

é n! / [k!(n-k)!]

onde n!=n(n-1)(n-2)...3.2.1

Era isso que vc queria?

Bruno Leite

At 22:34 01/12/01 -0300, you wrote:
>gostaria de saver se existe alguma fórmula para
>determinar qualqier termo do triângulo de pascal sem
>ter que montá-lo.
>
>___
>Yahoo! GeoCities
>Tenha seu lugar na Web. Construa hoje mesmo sua home page no Yahoo! 
>GeoCities. É fácil e grátis!
>http://br.geocities.yahoo.com/




Re: Potência infinita ?

2001-12-09 Thread Bruno F. C. Leite

At 11:28 09/12/01 -0200, you wrote:

>- Original Message -
>From: "Marcelo Souza" <[EMAIL PROTECTED]>
>To: <[EMAIL PROTECTED]>
>Sent: Sunday, December 09, 2001 9:10 AM
>Subject: Re: Potência infinita ?
>
>
> > Quer ter seu próprio endereço na Internet?
> > Garanta já o seu e ainda ganhe cinco e-mails personalizados.
> > DomíniosBOL - http://dominios.bol.com.br
> >
> >
> >
> >
> >
> > Desculpa, não tinha lido a essencia da sua pergunta.
> >
> > vejamos x^x^...=k => x^k=k => x=raiz k-ésima de k
> > Se vc mudar dois por quatro, vc na realidade, não mudou absolutamente
>nada,
> > visto que sqrt2 é o mesmo que raiz quarta de 4. Teoricamente, se vc ir
> > substituindo k por números 3,4,5,6...cada vez terá um número menor para x,
> > pois a sequencia 1, sqrt2,raiz cubica de 3,
> > raiz quarta de 4...e assim por diante é decrescente a partir do terceiro
> > termo. Isto é fácil de se provar por indução. Há um equivoco em dizer que
>a
> > potencia infinita pode valer 2 ou 4...A potencia infinita pode valer
>quanto
> > o cara que montou o problema quiser  . Depende do que há do outro lado da
> > igualdade. Isso determina o valor da potencia infinita. O fato de ser 2 ou
>4
> > implica que valem, na realidade a mesma coisa o valor de x. O que há com o
> > seu raciocinio, pelo menos é o que eu estou achando, é uma confusão de
> > incognitas.
> > se eu disser
> > x^x^...=2 e x^x^...=4 aí é um absurdo, pois estou afirmando que 2=4...Vc
> > deve estar confundindo as incognitas.
> > A resposta disso é sempre x=raiz k-esima de k para x^x^x^...=k
> > ok?
> > Qquer equivoco, me perdoe
> > Um abraço
> > Marcelo
> > >From: [EMAIL PROTECTED]
> > >Reply-To: [EMAIL PROTECTED]
> > >To: <[EMAIL PROTECTED]>
> > >Subject: Potência infinita ?
> > >Date: Sun, 09 Dec 2001 01:49:34 EST
> > >
> > >   Olá colegas da lista,
> > >   Vi uma resolução de uma interessante questão com potências que muito
> > >depois já não me parece correta. Quero saber se há algum erro. Obrigado.
> > >   Trata-se de uma incógnita que está eleva a ela mesma infinitas vezes
> > >(sem parênteses) igual a dois, isto é, x elevado a x, que este está
>elevado
> > >a x ... igual a dois. A solução vem da percepção de que pode-se esquecer
>do
> > >primeiro x (da base) e substituir o resto por dois. Têm se assim x ao
> > >quadrado igual a dois. Até aí há algum erro ? O x pode valer + ou - a
>raiz
> > >de dois ?
> > >   O que me faz parecer que há um erro é que se mudarmos o problema
> > >trocando o dois por quatro, a solução permanece a mesma. Assim essa
> > >"potência infinita" seria algo indeterminado pois pode valer dois ou
> > >quatro.
> > >   Obrigado pela atenção.
> > >  Raul
> >
> >
> > _
> > Get your FREE download of MSN Explorer at http://explorer.msn.com/intl.asp
> >
> >
>
>Na realidade só há sentido discutir as soluções para x^x^x^x=k, quando
>k<= e. Para maiores detalhes consultar a RPM 26 no artigo "Perigos" da
>Profissão do Prof. Vicenzo Bongiovanni. Um abraço a todos,
>Poncio Mineiro

Só completando: você só poderia substituir os "andares" a partir do segundo 
por 4 se de fato existisse x tal que a equação x^x...=4 tem solução. Daí 
vem o erro.

Bruno Leite




Re: Teoria dos números

2001-12-09 Thread Bruno F. C. Leite

At 11:41 09/12/01 -0500, you wrote:
>   Olá colegas,
>   obrigado pela atenção na questão de potências e, relativo a ela, onde 
> encontro a RPM 26 ?
>   Agora, teve uma questão do IME que um aluno me mostrou e só sei 
> resolver usando o pequeno teorema de Fermat, gostaria de saber se há 
> outra resolução.
>   Trata-se de provar que K e K^5 terminam com o mesmo algarismo para todo 
> K inteiro.
>   Prova-se usando que K^5 - K é divisível por dois e por 5 (onde se usa o 
> pequeno teorema).
>   Acho que pode haver outra resolução pois não acredito que o IME queria 
> cobrar o pequeno teorema de Fermat, ou será possível ?
>   Obrigado pela atenção,
>  Raul

Se k é ímpar, k^5 é impar. Se k é par, k^5 é par. Logo k^5-k é par sempre. 
Agora vamos mostrar que k^5-k é multiplo de 5. Divida  k por 5. teremos 
k=5a+b com 0<=b<=4. Então k^5-k=k(k^4-1)=(5a+b)[(5a+b)^4-1]. fazendo as 
contas e testando os 5 valores possiveis para b, temos que k^5-k é múltiplo 
de 5. (há várias outras maneiras de fazer isso sem usar teorema de Fermat)

Bruno Leite




Re: Teoria dos números

2001-12-09 Thread Bruno F. C. Leite

At 16:17 09/12/01 -0200, you wrote:
>At 11:41 09/12/01 -0500, you wrote:
>>   Olá colegas,
>>   obrigado pela atenção na questão de potências e, relativo a ela, onde 
>> encontro a RPM 26 ?
>>   Agora, teve uma questão do IME que um aluno me mostrou e só sei 
>> resolver usando o pequeno teorema de Fermat, gostaria de saber se há 
>> outra resolução.
>>   Trata-se de provar que K e K^5 terminam com o mesmo algarismo para 
>> todo K inteiro.
>>   Prova-se usando que K^5 - K é divisível por dois e por 5 (onde se usa 
>> o pequeno teorema).
>>   Acho que pode haver outra resolução pois não acredito que o IME queria 
>> cobrar o pequeno teorema de Fermat, ou será possível ?
>>   Obrigado pela atenção,
>>  Raul
>
>Se k é ímpar, k^5 é impar. Se k é par, k^5 é par. Logo k^5-k é par sempre. 
>Agora vamos mostrar que k^5-k é multiplo de 5. Divida  k por 5. teremos 
>k=5a+b com 0<=b<=4. Então k^5-k=k(k^4-1)=(5a+b)[(5a+b)^4-1]. fazendo as 
>contas e testando os 5 valores possiveis para b, temos que k^5-k é 
>múltiplo de 5. (há várias outras maneiras de fazer isso sem usar teorema 
>de Fermat)
>
>Bruno Leite


pensei em outra maneira de resolver isto. Temos que k(k^4-1) é múltiplo de 
5 se e só se k(k^4-5k^2+4) é multiplo de 5. mas 
k(k^4-5k^2+4)=k(k^2-1)(k^2-4)=(k-2)(k-1)k(k+1)(k+2), e o produto de 5 
números consecutivos é múltipo de 5!

Bruno Leite







Re: Completude da Geometria e Teorema de Godel

2001-12-14 Thread Bruno F. C. Leite

At 17:56 14/12/01 -0300, you wrote:
>vc disse sobre as propriedades do sistema formal e
>sobre a consistencia e a completude.Como vc encara o
>antagonismo das duas últimas???Vc apenas sabe o que
>Godel provou ou ENTENDE BEM o que ele demonstrou???è
>uma coisa de fácil entendimento como 2+2=4,ou ele
>demonstrou de forma dificil de se entender e vc só
>memorizou o resultado??Vc está entendendo o que quero
>dizer??O que quero falar se isso é uma coisa clara
>,lógica ,que está na cara ,ou um resultado avançado.

Metendo-me na conversa:

Acho que o teorema de Gödel não é difícil de se entender (talvez entender 
bem o papel da "aritmética de Peano" no enunciado seja o mais confuso), mas 
a demonstração é bem complicada.

Bruno Leite


>--- Paulo Santa Rita <[EMAIL PROTECTED]> escreveu: >
>Ola Rogerio e demais
> > colegas desta lista,
> >
> > E importante que se compreenda corretamente o que e
> > um SISTEMA FORMAL e o
> > que vem a ser COMPLETUDE e CONSISTENCIA num tal
> > sistema. Estes sistemas tem,
> > a grosso modo :
> >
> > 1) Objetos indefinidos ( ou primitivos )
> > 2) proposicoes primitivas ( ou axiomas, ou
> > postulados )
> >
> > NAO SE PODE ATRIBUIR AOS OBJETOS PRIMITIVOS NENHUMA
> > PROPRIEDADE DITADA POR
> > UMA EVENTUAL REPRESENTACAO MENTAL E INTUITIVA QUE
> > TENHAMOS DELES. Tudo que
> > se falar sobre os objetos deve ser uma consequencia
> > logica dos axiomas e dos
> > teoremas que ja tenhamos demonstrado. Pode-se
> > construir novos objetos em
> > estrita obediencia as regras de construcao.
> >
> > 1)Um sistema formal e CONSISTENTE se nao for
> > possivel provar uma afirmacao e
> > a sua negacao, isto e, exemplificando, se eu provar
> > que "A e B" eu nao
> > poderei provar que "A e nao B"
> >
> > 2)Um sistema formal e COMPLETO se todas as
> > afirmacoes sobre os objetos puder
> > ser provada com os recursos de inferencia do proprio
> > sistema, isto e, nao
> > pode haver uma propriedade usufruida por alguns
> > objetos do sistema que seja
> > indemonstravel com os recursos de inferencia do
> > sistema.
> >
> > Em geral, criar uma sistema formal e, em geral, um
> > dos objetivos perseguidos
> > para qualquer ramo da matematica, sobretudo quando
> > ele ja esta
> > suficientemente maduro e ja deu bons resultados.
> >
> > A grosso modo, o que Godel mostrou e que os dois
> > conceitos acima, de
> > COMPLETUDE e INCONSISTENCIA, sao antagonicos para
> > qualquer sistema formal
> > que use minimos recursos da Aritmetica, isto e :
> >
> > "Se o sistema formal for COMPLETO, isto e, toda
> > afirmacao sobre os objetos
> > do sistema puderem ser demonstradas com os recursos
> > de inferencia do
> > sistema, entao ele sera INCONSISTENTE, vale dizer,
> > nos seremos capazes de
> > provar uma teorema e a negacao dele; Se, por outro
> > lado, o sistema formal
> > for CONSISTENTE, isto e, se nunca poder acontecer de
> > provarmos um teorema e
> > a sua negacao, entao ele sera INCOMPLETO, vale
> > dizer, haverao propriedades
> > validas dos objetos do sistema que nos nao seremos
> > capazes de provar com os
> > recursos de inferencia do proprio sistema."
> >
> > Nao existe Teorema da Completude na Geometria
> > Euclidiana. Nao no sentido de
> > COMPLETUDE de um sistema formal. Hilbert mostrou que
> > a geometria euclidiana
> > seria consistente, se a algebra tambem fosse. Mas a
> > consistencia da Algebra
> > depende da Aritmetica e a prova da consistencia
> > desta ultima parece muito
> > dificil de ser conseguida ...
> >
> > Ate parece, numa primeira apreciacao, que o Teorema
> > de Godel e algo ruim e
> > negativo... Ele sulapou o sonho de Hilbert e de
> > todos os Matematicos
> > formalistas, que com seus sistemas formais, tiravam
> > o sentido intuitivo que
> > damos aos objetos matematicos, reduzindo a
> > Matematica a um jogo logico sem
> > graca, sem semantica e sem sentido.
> >
> > Observe que COMPLETUDE e CONSISTENCIA sao
> > propriedade DO SISTEMA FORMAL, nao
> > de um de seus objetos : sao portanto propriedades do
> > TODO. Visto por este
> > angulo, Godel mostrou que o TODO tem propriedades (
> > consistencia, completude
> > )  que sao inacessiveis ou inesplicaveis pela mera
> > consideracao das partes
> > que o compoe O TODO, isto e, O TODO E MAIS QUE A
> > MERA SOMA DAS PARTES. O
> > cara formalista pressupoe justamente o contrario.
> > Ele pensa que conhecendo
> > bem as partes ( axiomas, teoremas, objetos
> > indefinidos ) vai poder explicar
> > ( demonstrar ) tudo que aparecer ou ocorrer na
> > frente dele. E o SONHO
> > EISNTENIANO de encontrar UM CONJUNTO DE EQUACOES QUE
> > EXPLICAM TODO O
> > UNIVERSO.
> >
> > Godel, nos permitiu comecar a pensar NO SENTIDO, NA
> > SEMANTICA, NO FIM, NA
> > FUNCAO, NO PAPEL, NA INTERPRETACAO TELEOLOGICA, como
> > algo mais que mera
> > filosofia barata. Se se retirar o sentido das
> > coisas, as coisa perdem o
> > sentido. Agora, como articular de forma consistente
> > e seria este sentido ?
> >
> > Todos os danos que estamos causando ao mundo
> > natura

Re: beal

2001-12-17 Thread Bruno F. C. Leite

At 13:20 17/12/01 +, you wrote:

>No livro "Filosofia da Matemática", de Stephen Barker, li uma comparação 
>muito interessante para explicar o que é o princípio da indução. Ele 
>compara os números naturais com uma fila infinita de peças de dominó 
>colocadas em pé. Se derrubarmos a primeira peça e, se soubermos que cada 
>peça, ao cair, derrubará a seguinte, saberemos que todas serão derrubadas.
>
>Agora, quanto à conjectura de Beal, nunca ouvi falar. Aliás, nunca ouvi 
>falar de nenhum matemático chamado Beal. Alguém sabe algo sobre ele?

Não tem nem no http://turnbull.mcs.st-and.ac.uk/~history/Mathematicians/

É assim mesmo que se escreve?

Bruno




>>From: "Marcelo Souza" <[EMAIL PROTECTED]>
>>Reply-To: [EMAIL PROTECTED]
>>To: [EMAIL PROTECTED]
>>Subject: Re: beal
>>Date: Sun, 16 Dec 2001 20:37:23 +
>>
>>2) Vc quer aprender indução, é isso? Eu acho que o artigo do Elon da revista
>>Eureka é uma boa pedida para um treino assim como para um aprendizado, está
>>bem explicado, não está confuso...É bom ler, mas é melhor ainda ter certeza
>>do que se pode fazer com indução.
>>O princípio da indução diz, basicamente, que, dada uma propriedade S(n)
>>válida para um número n natural. Se S(1) é válida e, se o fato de S(K) valer
>>implicar que S(K+1) vale, então, S vale para todos os naturais.
>>Vejamos um exemplo simples:
>>Mostre que 1+2+3+...+n = [n(n+1)]/2
>>Primeiro passo: Ver se vale para n=1
>>1=1(2)/2 =1 (0K)
>>Segundo: Assuma que vale para K e tente provar para K+1
>>Se vale para K então
>>
>>1+2+...+k = k(k+1)/2
>>Vc quer provar para k+1, certo? Logo, o lado esquerdo está precisando de
>>somar k+1, para não alterar, somar dos dois lados
>>1+2+...+k+(k+1)=k(k+1)/2 + (k+1)
>>   = (k+1)(k+2)/2
>>Isto prova que vale para k+1, pois note que é a mesma fórmula de k, mas com
>>k+1 ao invés de k.
>>Faça como exercício esta
>>Mostrar que 1+2^2+3^2+...+n^2=n(n+1)(2n+1)/6
>>Ok
>>valeu
>>Marcelo
>>
>>
>>>From: "gabriel guedes" <[EMAIL PROTECTED]>
>>>Reply-To: [EMAIL PROTECTED]
>>>To: <[EMAIL PROTECTED]>
>>>Subject: beal
>>>Date: Sat, 15 Dec 2001 18:46:37 -0200
>>>
>>>tudo bem colegas da lista,
>>>1)Alguem ja ouviu  falar na conjectura de beal oque que ela propõe e etc???
>>>
>>>2)Estava dando uma olhada em indução finita , e queria me a profundar
>>>,alguem conhece um bom livro  ?
>>
>>
>>_
>>MSN Photos is the easiest way to share and print your photos:
>>http://photos.msn.com/support/worldwide.aspx
>
>
>_
>Join the world’s largest e-mail service with MSN Hotmail. 
>http://www.hotmail.com
>




Re: beal

2001-12-17 Thread Bruno F. C. Leite

Ahá!

http://www.bealconjecture.com/

http://primes.utm.edu/glossary/page.php/BealsConjecture.html

Bruno Leite


At 17:16 17/12/01 -0200, you wrote:
>Pelo amor de Deus, não consigo dormir de curiosidade. Sobre qual assunto é 
>essa conjectura de Beal? Internautas, ninguém descobriu nada sobre esse cara?
>
>Bruno F. C. Leite wrote:
>
>>At 13:20 17/12/01 +, you wrote:
>>
>>>No livro "Filosofia da Matemática", de Stephen Barker, li uma comparação 
>>>muito interessante para explicar o que é o princípio da indução. Ele 
>>>compara os números naturais com uma fila infinita de peças de dominó 
>>>colocadas em pé. Se derrubarmos a primeira peça e, se soubermos que cada 
>>>peça, ao cair, derrubará a seguinte, saberemos que todas serão derrubadas.
>>>
>>>Agora, quanto à conjectura de Beal, nunca ouvi falar. Aliás, nunca ouvi 
>>>falar de nenhum matemático chamado Beal. Alguém sabe algo sobre ele?
>>
>>
>>Não tem nem no http://turnbull.mcs.st-and.ac.uk/~history/Mathematicians/
>>
>>É assim mesmo que se escreve?
>>
>>Bruno
>>
>>
>>
>>
>>>>From: "Marcelo Souza" <[EMAIL PROTECTED]>
>>>>Reply-To: [EMAIL PROTECTED]
>>>>To: [EMAIL PROTECTED]
>>>>Subject: Re: beal
>>>>Date: Sun, 16 Dec 2001 20:37:23 +
>>>>
>>>>2) Vc quer aprender indução, é isso? Eu acho que o artigo do Elon da 
>>>>revista
>>>>Eureka é uma boa pedida para um treino assim como para um aprendizado, está
>>>>bem explicado, não está confuso...É bom ler, mas é melhor ainda ter certeza
>>>>do que se pode fazer com indução.
>>>>O princípio da indução diz, basicamente, que, dada uma propriedade S(n)
>>>>válida para um número n natural. Se S(1) é válida e, se o fato de S(K) 
>>>>valer
>>>>implicar que S(K+1) vale, então, S vale para todos os naturais.
>>>>Vejamos um exemplo simples:
>>>>Mostre que 1+2+3+...+n = [n(n+1)]/2
>>>>Primeiro passo: Ver se vale para n=1
>>>>1=1(2)/2 =1 (0K)
>>>>Segundo: Assuma que vale para K e tente provar para K+1
>>>>Se vale para K então
>>>>
>>>>1+2+...+k = k(k+1)/2
>>>>Vc quer provar para k+1, certo? Logo, o lado esquerdo está precisando de
>>>>somar k+1, para não alterar, somar dos dois lados
>>>>1+2+...+k+(k+1)=k(k+1)/2 + (k+1)
>>>>= (k+1)(k+2)/2
>>>>Isto prova que vale para k+1, pois note que é a mesma fórmula de k, mas com
>>>>k+1 ao invés de k.
>>>>Faça como exercício esta
>>>>Mostrar que 1+2^2+3^2+...+n^2=n(n+1)(2n+1)/6
>>>>Ok
>>>>valeu
>>>>Marcelo
>>>>
>>>>
>>>>>From: "gabriel guedes" <[EMAIL PROTECTED]>
>>>>>Reply-To: [EMAIL PROTECTED]
>>>>>To: <[EMAIL PROTECTED]>
>>>>>Subject: beal
>>>>>Date: Sat, 15 Dec 2001 18:46:37 -0200
>>>>>
>>>>>tudo bem colegas da lista,
>>>>>1)Alguem ja ouviu falar na conjectura de beal oque que ela propõe e etc???
>>>>>
>>>>>2)Estava dando uma olhada em indução finita , e queria me a profundar
>>>>>,alguem conhece um bom livro ?
>>>>
>>>>
>>>>
>>>>_
>>>>MSN Photos is the easiest way to share and print your photos:
>>>>http://photos.msn.com/support/worldwide.aspx
>>>
>>>
>>>
>>>_
>>>Join the world's largest e-mail service with MSN Hotmail. 
>>>http://www.hotmail.com
>>
>
>




Re: universitario

2001-12-20 Thread Bruno F. C. Leite

O Brasil não participa da mundial universitária?

Bruno

At 11:57 20/12/01 +, you wrote:

>  Tem a mundial universitária, IMC.
>
>
>
>
>
>>From: "gabriel guedes" <[EMAIL PROTECTED]>
>>Reply-To: [EMAIL PROTECTED]
>>To: <[EMAIL PROTECTED]>
>>Subject: universitario
>>Date: Mon, 17 Dec 2001 19:32:40 -0200
>>
>>Ola amigos,
>>Alem da obm e da ibero existe alguma  competição importante no nivel 
>>universitario ( q brasileiros possam participar)?
>>E pra quem ja é formado tem alguma coisa ou so resta tentar resolver  um 
>>dos problemas do claymath?
>
>
>_
>Get your FREE download of MSN Explorer at http://explorer.msn.com/intl.asp.
>




Re: Dúvida

2001-12-22 Thread Bruno F. C. Leite

At 14:51 22/12/01 -0300, you wrote:

>Olá colegas da lista,
>
>
>
>Vi no cursinho a seguinte questão:
>
>
>
>Sejam x, y e z números reais positivos.
>
>a)   Mostre que (x+1/x)*(y+1/y)*(z+1/z) >= 8

É fácil provar na marra (só com manipulações simples) que se x>0, x+1/x>=2 
(e isso acaba com o problema a) )


>b)   Mostre que, se x*y*z=100, então (x+1)*(y+1)*(z+1) >= 80
>
>
>
>Meu professor resolveu a questão com a idéia de que a média aritmética
>
>de dois números reais positivos é sempre maior ou igual a média geométrica.
>
>
>
>Haveria uma outra forma de resolução, sem ter que tirar esta informação (das
>
>médias) da manga?
>
>
>
>O item b) não é meio estranho? Se x, y e z são reais positivos e xyz já é
>
>100, parece meio óbvio a demonstração, já que, ao desenvolver aquele
>
>produto, haverá uma soma de xyz (=100) e outros termos todos positivos...
>
>Em que eu estou errando ao fazer este raciocínio?
>
>
>
>Não é o mesmo que: Seja abc = 10. Prove que (abc + 2) >= 5 ?

Você tem razão! mas um problema parecido e não óbvio seria

x*y*z=1, então (x+1)*(y+1)*(z+1) >= 8

Bruno Leite


>
>
>
>
>Valeu
>
>




Re: Semana Olímpica

2001-12-25 Thread Bruno F. C. Leite

At 18:19 23/12/01 -0200, you wrote:
>Olá Pessoal,
>
>Gostaria de saber se vai ter muita gente do nível universitário na Semana
>Olímpica. Receio chegar lá e ser o único universitário...

Eu vou tb!

Bruno Leite


>Até mais
>
>[ Vinicius José Fortuna  ]
>[ [EMAIL PROTECTED] ]
>[  Visite www.viniciusf.cjb.net  ]




Re: Semana Olimpica 2002

2001-12-28 Thread Bruno F. C. Leite

Ah, é verdade, eu tb estou curioso.

Bruno Leite

At 17:03 28/12/01 -0200, you wrote:
>Saudações.
>
>Alguém pode dizer qual será o programa da Semana Olímpica de 2002? Só pra
>ter uma idéia de como vai ser.
>
>Obrigado,
>Bernardo
>
>-- Mensagem original --
>
> >Gostaria de chamar a atenção de todos para a possibilidade de um pequeno
> >número de alunos que ganharam menção honrosa, ou até que não ganharam nada,
> >participarem da Semana Olímpica 2002. A mensagem abaixo está na seção de'
> >novidades na home page da OBM. Ou para quem preferir ir direto, está em
> >http://www.obm.org.br/semana2002.html
> >
> >[]s, N.
> >
> >= 
> 
> >
> >Os premiados com medalhas na OBM 2001 estão todos convidados a participar
> >na Semana Olímpica 2002, de 4 a 11 de janeiro de 2002 em Vitória, ES.
> >
> >Há um número limitado de vagas para alunos que ganharam menção honrosa.
> >Caso estas vagas não sejam preenchidas, alunos sem nenhum premio também
> >poderão participar.
> >Será cobrada uma taxa de hospedagem e alimentação dos alunos que não 
> ganharam
> >medalha na OBM 2001.
> >
> >Tendo ganhado medalha, menção honrosa ou nenhum prêmio,
> >os interessados devem entrar em contato com a secretaria da OBM com 
> urgência.
> >
> >
>
>
>
>--
>Use o melhor sistema de busca da Internet
>Radar UOL - http://www.radaruol.com.br




OIMU

2001-12-30 Thread Bruno F. C. Leite

Alguém tem uma previsão de quando sai os resultados da OIMU?

Bruno Leite




Re: Onde compro esses livros?

2001-12-30 Thread Bruno F. C. Leite

At 16:38 30/12/01 -0200, you wrote:
>Eu estava procurando os livros "Problem-Solving Strategies", "Winning 
>Solutions" e um outro que era uma compilação de IMO's. Só achei no 
>Amazon.com e outras livrarias norte-americanas; obviamente, é caríssimo e
>demora muito para chegar.

Em livcultura.com.br vc deve achar os livros, mas são bem caros, e demoram 
muito para chegar se eles não tiverem no estoque.

Bruno Leite


>  Alguem sabe em que outro lugar (de preferência no Brasil, claro) eu 
> posso achar esses livros ?
>
>[]s, Jorge Peixoto




Re: ???

2001-12-31 Thread Bruno F. C. Leite

At 01:08 01/01/02 -0200, you wrote:
>Quantos números de 1 a 1998 podem ser escritos como a soma de duas ou mais 
>potências de 3?

Será que eu entendi direito? Tome x natural, com 1<=x<=1998. Escreva x na 
base 3, e teremos
x=a_0 * 3^0 + a_1 * 3^1 + ... + a_n * 3^n, com a_i = 0, 1 ou 2. Se x não 
for potência de 3, isso é uma forma de escrever x como soma de 2 ou mais 
potências de 3. Se x=3^n, com n>0, então x=3*3^(n-1).

A solução é "todos os números, menos 1".

outra forma: divida x por 3: x=3a+b=a*3^1+b*3^0...

Bruno Leite





Re: Por favor, me tirem dessa lista.

2002-01-09 Thread Bruno F. C. Leite

At 15:56 09/01/02 +, you wrote:


>Por favor, alguem me informe como me excluo dessa lista.

É uma operação muito complexa. Está detalhado em 
http://www.obm.org.br/lista.htm

Bruno

>Grato Tomas
>MailBR - O e-mail do Brasil -- http://www.mailbr.com.br
>Faça já o seu. É gratuito!!!




Re: Somatório dos primeiros impares

2002-01-11 Thread Bruno F. C. Leite

At 22:59 10/01/02 -0200, you wrote:
>Amigos,
>
>Li sobre uma regra de Pitágoras para se calcular a soma dos n primeiros
>números impares, por n^2.
>Ex: A soma dos 9 primeiros números impares é 9^2 = 81.
>Achei interessante a simplicidade da "fórmula".. Tentei chegar a ela usando
>a formula da soma dos n numeros de uma PA, mas nao consegui, alguem pode me
>ajudar?

Há uma demonstração geométrica imediata. Considere os quadrados Q_n com 
coordenadas (0,0), (0,n), (n,0) e (n,n), e considere as fatias 
F_n=Q_{n+1}-Q_{n}
Veja que a fatia F_n tem (n+1)^2-n^2 = 2n+1 elementos, e que a união de 
F_0, F_1,...F_{n-1} dá Q_n, logo a soma dos n primeiros ímpares dá n^2. 
(com uma figura fica trivial...)

Bruno Leite





Re: [obm-l] Conjectura de Haeser

2002-01-25 Thread Bruno F. C. Leite

At 01:55 25/01/02 -0200, you wrote:
>Olá pessoal da lista.
>
>Tenho uma pequena conjectura a anunciar, não sei se ela já existe, nem se
>ela é verdadeira, mas aí vai :

Oi,

Isto já é conhecido...se vc pega um polinômio de grau n, digamos 
f(x)=x^n+...+a_1 x+a_0, então a diferença delta(f)=f(x+1)-f(x) é n vezes um 
polinômio monico de grau n-1.

Não é difícil provar isto, e nem é difícil ver que isto implica o que vc 
descobriu. Observe que ao fazer a diferença, vc reduz o grau do polinomio. 
Como vc quer a enésima diferença (delta de delta de delta...de delta de f) 
de um polinomio de grau n, só vai sobrar um termo de grau zero, que veio do 
x^n que aparece em f(x). Ou seja, o resto do polinomio original some no 
processo. Então podemos supor f(x)=x^n. Mas aí é claro que delta(f)= n 
vezes um polinômio monico de grau n-1. O resto segue fácil por indução. (ou 
poderíamos ter usado que delta (f+g)=delta (f)+delta(g) )

Você pode ve isto com mais detalhes num livro de diferenças finitas.

Bruno Leite
www.ime.usp.br/~brleite


>Dada uma sequência de n+1 potências consecutivas de n (1^n,2^n,..,(n+1)^n
>é um exemplo)
>
>faça a subtração dos termos consecutivos e teremos uma nova sequência, agora
>com n elementos:
>{(n+1)^n-n^n,n^n-(n-1)^n,2^n-1^n}
>
>repita o procedimento n vezes e obteremos apenas um número que é n! (n 
>fatorial)
>
>veja um exemplo :
>
>9³ - 8³ - 7³ - 6³ - 5³ - 4³ - 3³ - 2³ - 1³ - 0³
>__217__169__127__91___61___37___1971
>_48___42___36___30__24___1812___6
>666___666_6
>6=2*3=3!
>
>será que alguém poderia me ajudar a esclarecer ??
>Obrigado !
>
>"Mathematicus nascitur, non fit"
>Matemáticos não são feitos, eles nascem
>
>
>--
>Use o melhor sistema de busca da Internet
>Radar UOL - http://www.radaruol.com.br
>
>
>
>=
>Instruções para entrar na lista, sair da lista e usar a lista em
>http://www.mat.puc-rio.br/~nicolau/olimp/obm-l.html
>O administrador desta lista é <[EMAIL PROTECTED]>
>=

=
Instruções para entrar na lista, sair da lista e usar a lista em
http://www.mat.puc-rio.br/~nicolau/olimp/obm-l.html
O administrador desta lista é <[EMAIL PROTECTED]>
=



Re: [obm-l] Re: [obm-l] Conjectura de Haeser

2002-01-25 Thread Bruno F. C. Leite

Realmente, é bem interessante ver as relações entre cálculo discreto e 
contínuo, relações entre somas e integrais, etc. O livro Matemática 
Concreta (Knuth, Graham, Patashnik) fala disso em seu segundo capítulo. (e 
no nono, com a fórmula de Euler)

Lembro-me de que isto foi uma das coisas mais legais que estudei em 2000, 
pois o cálculo de somas terríveis ficava (trivialmente) reduzido ao 
problema de se achar antidiferenças de algumas funções.

Bruno Leite
www.ime.usp.br/~brleite


At 20:16 25/01/02 -0200, you wrote:
>Sauda,c~oes,
>
>Esta conjectura faz parte do estudo de PAs de ordem superior,
>Diferenças Finitas, polinômios fatoriais, antidiferenças, cálculo[
>de séries, recorrências etc.
>
>  Uma curiosidade:
>
>i) no mundo contínuo, d(e^x) = e^x ;
>
>ii) no mundo discreto, D(2^x) = 2^{x+1} - 2^x = 2^x.
>
>[]´s
>Luís (Rio de Jan.)
>>-Mensagem Original-
>>De: Augusto César Morgado
>>Para: [EMAIL PROTECTED]
>>Enviada em: sexta-feira, 25 de janeiro de 2002 14:29
>>Assunto: Re: [obm-l] Re: [obm-l] Conjectura de Haeser
>>
>>Ha uma analogia entre diferenças e derivadas. Basta trocar as potencias 
>>ordinarias por potencias fatoriais (potencia ordinaria x^3=x*x*x; 
>>potencia fatorial x^3=x*(x-1)*(x-2).
>>Leia o Richardson, An Introduction to (the?) Calculus of Finite 
>>Differences. Eh livro interessante e de facil leitura.
>>Morgado, Rio de Janeiro.

=
Instruções para entrar na lista, sair da lista e usar a lista em
http://www.mat.puc-rio.br/~nicolau/olimp/obm-l.html
O administrador desta lista é <[EMAIL PROTECTED]>
=



Fwd: Re: [obm-l] Re: [obm-l] Conjectura de Haeser

2002-01-25 Thread Bruno F. C. Leite


>Date: Fri, 25 Jan 2002 21:04:07 -0200
>To: [EMAIL PROTECTED]
>From: "Bruno F. C. Leite" <[EMAIL PROTECTED]>
>Subject: Re: [obm-l] Re: [obm-l] Conjectura de Haeser
>
>Realmente, é bem interessante ver as relações entre cálculo discreto e 
>contínuo, relações entre somas e integrais, etc. O livro Matemática 
>Concreta (Knuth, Graham, Patashnik) fala disso em seu segundo capítulo. (e 
>no nono, com a fórmula de Euler)
>
>Lembro-me de que isto foi uma das coisas mais legais que estudei em 2000, 
>pois o cálculo de somas terríveis ficava (trivialmente) reduzido ao 
>problema de se achar antidiferenças de algumas funções.
>
>Bruno Leite
>www.ime.usp.br/~brleite
>
>
>At 20:16 25/01/02 -0200, you wrote:
>>Sauda,c~oes,
>>
>>Esta conjectura faz parte do estudo de PAs de ordem superior,
>>Diferenças Finitas, polinômios fatoriais, antidiferenças, cálculo[
>>de séries, recorrências etc.
>>
>>  Uma curiosidade:
>>
>>i) no mundo contínuo, d(e^x) = e^x ;
>>
>>ii) no mundo discreto, D(2^x) = 2^{x+1} - 2^x = 2^x.
>>
>>[]´s
>>Luís (Rio de Jan.)

Acabei me esquecendo de dizer que, assim como calculamos a integral de x 
e^x por partes, calculamos a SOMA de x 2^x por partes também. Há muitas 
analogias entre as duas áreas!
Também ia me esquecendo de dizer que de fato o livro do Richardson é legal, 
tem todas estas coisas e é bem simples de ler. Não tenho muita certeza, mas 
acho que o livro é 100% elementar.

Bruno Leite (SP)
www.ime.usp.br/~brleite

>>>-Mensagem Original-
>>>De: <mailto:[EMAIL PROTECTED]>Augusto César Morgado
>>>Para: <mailto:[EMAIL PROTECTED]>[EMAIL PROTECTED]
>>>Enviada em: sexta-feira, 25 de janeiro de 2002 14:29
>>>Assunto: Re: [obm-l] Re: [obm-l] Conjectura de Haeser
>>>
>>>Ha uma analogia entre diferenças e derivadas. Basta trocar as potencias 
>>>ordinarias por potencias fatoriais (potencia ordinaria x^3=x*x*x; 
>>>potencia fatorial x^3=x*(x-1)*(x-2).
>>>Leia o Richardson, An Introduction to (the?) Calculus of Finite 
>>>Differences. Eh livro interessante e de facil leitura.
>>>Morgado, Rio de Janeiro.

=
Instruções para entrar na lista, sair da lista e usar a lista em
http://www.mat.puc-rio.br/~nicolau/olimp/obm-l.html
O administrador desta lista é <[EMAIL PROTECTED]>
=



Re: [obm-l] corredor

2002-01-25 Thread Bruno F. C. Leite

At 23:53 25/01/02 -0300, you wrote:
>Em um corredoe existem 900 armários numerados de 1 a
>900.Novecentas pessoas numeradas de 1 a 900 atravessam
>este corredor ,uma a uma, em ordem crescente de
>numeração.Cada pessoa deve reverter os armários que
>sAõ múltiplos de sua numeração.Por exemplo, a pessoa
>de número 4 deve mexer nor armários 4,8,12,16,20,etc,
>abrindo aqyeles que estÃo fechados e fechando aqueles
>que estão abertos.Ao final, quais armários estarão
>abertos e quais estarão fechados?

Vamos supor que inicialmente todos estão fechados. (é importante saber a 
situação inicial dos armários)
Um armário n estará aberto se e só se o número de pessoas que mexeram no 
armário n for impar.

O número de pessoas que mexem no armário n é o número de divisores de n. 
Logo, isto equivale a dizer que n tem um número ímpar de divisores.

(quais sao os números n com quantidade impar de divisores?)
...
...
...
...
...
Resposta: Os quadrados perfeitos!

Vamos provar isto. Seja n um número com quantidade impar de divisores. Seja 
d um divisor de n. Então n/d tb é divisor de n. Logo, NORMALMENTE os 
divisores vêm aos pares. Para que n tenha uma quantidade impar de 
divisores, um destes pares (d, n/d) deve obedecer d=n/d. Logo n=d^2, n é 
quadrado perfeito. (e é claro que vale a recíproca)

Bruno Leite
www.ime.usp.br/~brleite



>___
>Yahoo! GeoCities
>Tenha seu lugar na Web. Construa hoje mesmo sua home page no Yahoo! 
>GeoCities. É fácil e grátis!
>http://br.geocities.yahoo.com/
>=
>Instruções para entrar na lista, sair da lista e usar a lista em
>http://www.mat.puc-rio.br/~nicolau/olimp/obm-l.html
>O administrador desta lista é <[EMAIL PROTECTED]>
>=

=
Instruções para entrar na lista, sair da lista e usar a lista em
http://www.mat.puc-rio.br/~nicolau/olimp/obm-l.html
O administrador desta lista é <[EMAIL PROTECTED]>
=



Re: [obm-l] (sem assunto)

2002-01-26 Thread Bruno F. C. Leite

Oi,

Assim, com poucas informações, acho que é possível saber não a 
multiplicidade da raiz, mas sim a PARIDADE da multiplicidade da raiz.

Se o gráfico de f(x) tangencia o eixo x em x_0 sem cruzá-lo, então x_0 é 
raiz de multiplicidade par. Se cruzar, é de multiplicidade ímpar. Não é 
difícil provar isto, mas não vou provar pq não sei se é isso que vc quer.

Sobre a pergunta: "se cortar o eixo, a raiz é necessariamente simples?", a 
resposta é não. Basta pensar no exemplo f(x)=x^(2n+1), onde a raiz (zero) 
tem multiplicidade 2n+1.

Bruno Leite SP www.ime.usp.br/~brleite
#
PS: Eu coloquei na minha home page (o endereço está acima) os arquivos TEX, 
DVI, PS referentes ao que eu provei na semana olímpica (a irracionalidade 
de pi e o Postulado de Bertrand [prova de Erdos] )
Espero que seja útil!
#


At 23:33 26/01/02 -0500, you wrote:
>Imagine uma função polinomial de grau 5 por exemplo.e imagine que seu
>grafico tangencie o eixo x no ponto x0. Existe algum estudo que nos permita
>saber pelo grafico apenas qual a multiplicidade dessa raiz??. Se cortar o
>eixo x  a raiz e necessariamente simples, não é??
>Um abraço
> Crom
>=
>Instruções para entrar na lista, sair da lista e usar a lista em
>http://www.mat.puc-rio.br/~nicolau/olimp/obm-l.html
>O administrador desta lista é <[EMAIL PROTECTED]>
>=

=
Instruções para entrar na lista, sair da lista e usar a lista em
http://www.mat.puc-rio.br/~nicolau/olimp/obm-l.html
O administrador desta lista é <[EMAIL PROTECTED]>
=



Re: [obm-l] Teorema de Fermat

2002-01-27 Thread Bruno F. C. Leite

At 00:46 28/01/02 +, you wrote:

>Quais são as "raízes triviais" da função zeta?

Olá Rogério Godel Júnior,

A função zeta é definida inicialmente pela equação

zeta(s)=soma(1/n^s), n=1,2...infinito (s é um complexo)

Esta série converge se e só se a parte real de s é>1. No semiplano (z 
complexo | Re(z)>1} não é difícil ver que zeta(s) NUNCA se anula.

de fato, temos soma(mu(n)/n^s),n=1,2,...infinito = 1/zeta(s) !!!

(para saber o que é mu(n), consulte o email do Nicolau que está indo junto 
com este email...lá embaixo)

Lembro-me de que quando aprendi esta fórmula acima (donde segue que zeta 
nunca se anula) pensei que a hipótese de Riemann não fazia o menor sentido. 
Afinal, ela dia que os zeros não triviais (mas zeta não se anula!?) de 
zeta(s) têm parte real =1/2 (mas, se Re(s)=1/2, a série nem ao menos 
converge )

Mas é claro que eu estava errado. Pode-se estender a definição de zeta para 
todo o plano complexo (holomorfa, com um pólo em s=1) por continuação 
analítica, e agora sim a função zeta tem raízes e faz sentido falar de 
zeta(1/2+bi)...

Pode-se provar que vale o seguinte:

$\zeta(1-s)=2(2\pi)^{-s} \gamma(s)\cos(s\pi /2)\zeta(s)$

(onde gamma é aquela função que o professor de estatística usava, lembra? - 
a que "generaliza" o fatorial)

Se vc botar s=2n+1 (n>1 natural) na formula acima, vai descobri que 
zeta(-2)=zeta(-4)=zeta(-6)=...=0

Os inteiros pares negativos são chamados "zeros triviais" de zeta.

Infelizmente, vc não vai achar isso num livro de lógica modal... Eu acho 
melhor vc consultar o apostol de teoria analíica dos números...

Abração

Bruno Leite
www.ime.usp.br/~brleite  (a rede ime está fora do ar nesse fim de semana)


>>From: "Nicolau C. Saldanha" <[EMAIL PROTECTED]>
>>Reply-To: [EMAIL PROTECTED]
>>To: [EMAIL PROTECTED]
>>Subject: Re: [obm-l] Teorema de Fermat
>>Date: Wed, 23 Jan 2002 12:54:08 -0200
>>
>>On Wed, Jan 23, 2002 at 07:58:54AM -0500, [EMAIL PROTECTED] wrote:
>> > Muito obrigado Ralph pelos comentarios sobre o enigma( ex) de Fermat.
>> > No livro simon Singh chamava o enigma de santo graal da matemática,
>> > classificando-o como o mais dificil de todos os tempos. Se o parâmetro 
>> for os
>> > gênios que o tentaram resolver eu concordo, mas se fosse o tempo nâo. 
>> Lembra
>> > dos tres antigos problemas clássicos? . A quadratura do circulo, 
>> trissecção
>> > do angulo e duplicação do cubo( com régua e compasso), levaram mais de 
>> 1600
>> > anos , até mostrarem que são problemas insolúveis.Qual o melhor 
>> parâmetro pra
>> > julgar se este ou aquele problema é o mais dificil de todos os tempos? 
>> Existe
>> > algo , hoje em dia, em qualquer área, que substitua o ultimo teorema de
>> > fermat??
>>
>>Em termos de antiguidade, os campeões absolutos, vindos desde a antiguidade
>>e em aberto até hoje, são: o problema da existência de números perfeitos
>>ímpares e o da infinitude do número de números perfeitos pares.
>>
>>Lembro que um inteiro positivo n é perfeito se a soma dos divisores
>>inteiros positivos de n menores do que n for n.
>>Os menores números perfeitos são
>>
>>6 = 2^1 (2^2 - 1) = 1 + 2 + 3
>>   28 = 2^2 (2^3 - 1) = 1 + 2 + 4 + 7 + 14
>>  496 = 2^4 (2^5 - 1) = 1 + 2 + 4 + 8 + 16 + 31 + 62 + 124 + 248
>>
>>Não é difícil demostrar que n par é perfeito se e somente se n é na forma
>>n = 2^(p-1) (2^p - 1) onde Mp = 2^p - 1 é primo (um primo de Mersenne).
>>Ninguém sabe se existe algum número perfeito ímpar e ninguém sabe
>>demonstrar que existem infinitos primos de Mersenne (donde infinitos
>>números perfeitos pares).
>>
>>Este problema apesar de antigo não é considerado muito importante pela
>>maioria dos matemáticos. O problema em aberto em geral considerado mais
>>importante (mais importante até do que o último teorema de Fermat) é
>>a hipótese de Riemann generalizada. Bem, eu devo confessar que não entendo
>>tão bem assim pq a hipótese de Riemann é tão importante, não sei o
>>suficiente sobre as aplicações. Em todo caso a versão clássica
>>da hipótese de Riemann diz que as raízes (complexas)
>>não triviais da função zeta estão sobre a reta [(parte real de z) = 1/2].
>>
>>Uma versão elementar é a seguinte. Defina
>>
>>mu(n) = (-1)^k se n for o produto de k primos distintos;
>>   0se n for múltiplo de algum quadrado.
>>
>>Os primeiros valores de mu são
>>
>>n  1  2  3  4  5  6  7  8  9 10 11 12 13 14 15 16 17 18 19 20
>>mu(n)  1 -1 -1  0 -1  1 -1  0  0  1 -1  0 -1  1  1  0 -1  0 -1  0
>>
>>Defina M(n) como sendo a soma de mu(k), k = 1..n
>>
>>n  1  2  3  4  5  6  7  8  9 10 11 12 13 14 15 16 17 18 19 20
>>M(n)   1  0 -1 -1 -2 -1 -2 -2 -2 -1 -2 -2 -3 -2 -1 -1 -2 -2 -3 -3
>>
>>A hipótese de Riemann consiste em afirmar que para todo a > 1/2 temos
>>
>> M(n)
>> lim     = 0
>>  n -> infty n^a
>>
>>
>>[]s, N.
>>=
>>Instruções para entrar na lista, sair da lista e usar a lista em
>>http://www.mat.pu

Re: [obm-l] Teorema de Fermat

2002-01-28 Thread Bruno F. C. Leite

At 21:27 28/01/02 +, you wrote:
>Como? zeta(-2)=1^2+2^2+3^2+4^2+...=0 ??

Sim, é isso mesmo, não é surpreendente?





É brincadeira! Isto está errado!!!

A série zeta(s)=soma(1/n^s), n=1,2...infinito (s é um complexo) SÓ CONVERGE 
PARA Re(s)>1, LOGO SÓ DEFINE UMA FUNÇÃO PARA Re(s)>1!!!

Já vou explicar isto melhor.

>Isso nem com a lógica paraconsistente consigo entender!!!
>Pode me explicar o que vc quer dizer com essa "extensão para todo o plano 
>complexo" (desculpe minha ignorância em teoria dos números e/ou análise 
>complexa, mas não sei o que é holomorfa, pólo nem continuação analítica). 
>Se a fórmula de zeta é a fórmula q vc mencionou, e a exponenciação por 
>complexo é a que eu conheço (expansão pelo polinômio de Taylor da função 
>e^x, não consigo imaginar nenhuma raíz "trivial". O q me parece imediato é 
>que não possui raiz real (a parte imaginária não pode ser nula) posi sei 
>que a função a^x (a>0) não tem raiz real.

Imagine uma função f:(disco unitário aberto de R^2) -> R, derivável, 
digamos f(x,y)=x+10. Veja que podemos estender esta função de vários modos 
(e deixando a extensão ainda derivável) para um domínio maior, digamos R^2. 
Por exemplo, podemos definir g:R^2->R por g(x,y)=x+10, mas há vários outros 
jeitos, do tipo

g(x,y)=x+10 se x<=1
g(x,y)=x^2/2+10,5 se x>1

Quando trocamos funções R^2->R por funções de C em C, isto nao acontece. 
Por exemplo, se temos uma função do disco unitário aberto do plano complexo 
em C que é HOLOMORFA (holomorfa=derivável) SÓ HÁ UM MODO DE ESTENDÊ-LA PARA 
UMA FUNÇÃO HOLOMORFA DE DOMÍNIO MAIOR. (um outro exemplo: se f é holomorfa 
e conhecemos f na fronteira de um círculo, f já está determinada dentro do 
círculo. - o que é fantástico, aliás...)

Veja, temos uma função (zeta) definida para Re(s)>1, ok? Ela é holomorfa no 
semiplano {z complexo | Re(z)>1}, e pode ser estendida DE UMA ÚNICA FORMA 
para uma função holomorfa no plano todo, digamos rogerio(s). Isso é a 
continuação analítica!!! [na verdade ela não vai ser analítica no plano 
todo, ela vai ter um ponto em que ela "explode" - UM POLO - em s=1]

Se re(s)>1, rogerio(s)=zeta(s)=soma(1/n^s), certo?
E se re(s)<1?? Embora a série de zeta não faça sentido, a função 
rogerio(s) está definida!!!
Oras, então DEFINIMOS, para re(s)<1, zeta(s)=rogerio(s).

Só para ser o mais repetitivo possível: zeta(s) só coincide com a série 
soma (1/n^s)  se Re(s)>1
Logo zeta(-2) NAO É e NEM PODERIA SER 1^2+2^2+3^2...

Ah, e a exponenciação com complexos de fato é a da série de Taylor, mas é 
mais fácil pensar em exp(a+bi)=e^a(cos b +i sen b). Como vc falou, é óbvio 
que zeta não tem raízes nos reais >1, mas não é TAO fácil ver que ela não 
se anula em todo o semiplano re(s)>1.

Espero ter deixado as coisas mais claras, assim como espero não ter dito 
nenhuma asneira!

Bruno Leite
www.ime.usp.br/~brleite


>Outro abração,
>  Rogério
>
>>From: "Bruno F. C. Leite" <[EMAIL PROTECTED]>
>>Reply-To: [EMAIL PROTECTED]
>>To: [EMAIL PROTECTED]
>>Subject: Re: [obm-l] Teorema de Fermat
>>Date: Mon, 28 Jan 2002 03:04:31 -0200
>>
>>At 00:46 28/01/02 +, you wrote:
>>
>>>Quais são as "raízes triviais" da função zeta?
>>
>>Olá Rogério Godel Júnior,
>>
>>A função zeta é definida inicialmente pela equação
>>
>>zeta(s)=soma(1/n^s), n=1,2...infinito (s é um complexo)
>>
>>Esta série converge se e só se a parte real de s é>1. No semiplano (z
>>complexo | Re(z)>1} não é difícil ver que zeta(s) NUNCA se anula.
>>
>>de fato, temos soma(mu(n)/n^s),n=1,2,...infinito = 1/zeta(s) !!!
>>
>>(para saber o que é mu(n), consulte o email do Nicolau que está indo junto
>>com este email...lá embaixo)
>>
>>Lembro-me de que quando aprendi esta fórmula acima (donde segue que zeta
>>nunca se anula) pensei que a hipótese de Riemann não fazia o menor sentido.
>>Afinal, ela dia que os zeros não triviais (mas zeta não se anula!?) de
>>zeta(s) têm parte real =1/2 (mas, se Re(s)=1/2, a série nem ao menos
>>converge )
>>
>>Mas é claro que eu estava errado. Pode-se estender a definição de zeta para
>>todo o plano complexo (holomorfa, com um pólo em s=1) por continuação
>>analítica, e agora sim a função zeta tem raízes e faz sentido falar de
>>zeta(1/2+bi)...
>>
>>Pode-se provar que vale o seguinte:
>>
>>$\zeta(1-s)=2(2\pi)^{-s} \gamma(s)\cos(s\pi /2)\zeta(s)$
>>
>>(onde gamma é aquela função que o professor de estatística usava, lembra? -
>>a que "generaliza" o fatorial)
>>
>>Se vc botar s=2n+1 (n>1 natural) na formula acima, vai descobri que
>>zeta(-2)=zeta(-4)=zeta(-6)=...=0
>>
>>Os inteiros pares

Re: [obm-l] Teorema de Fermat

2002-01-30 Thread Bruno F. C. Leite

At 18:30 30/01/02 +, you wrote:

>Me deixa eu ver se entendi. A função zeta(s) NÃO é  soma(1/n^s),  senão 
>ela não estaria definida para todo s complexo. Mas ela é uma extensão de 
>soma(1/n^s) onde está definida, para todo plano complexo. É isso? Nós 
>vamos estudar isso em funções analíticas?

Não sei bem o que vamos ver no curso de funções analíticas, mas acho que 
não se fala da função zeta.

>  Isso (a hipótese de Riemann) me parece mais um problema de análise do 
> que de teoria dos números. Por que é considerado teoria dos números?

Porque sim.

Bruno




Ok, vou falar sério. Euler foi o primeiro a ver uma ligação entre a função 
zeta e a teoria dos números, quando ele achou a fatoração "mágica" abaixo: 
(para re(s)>1, obviamente)

zeta(s)=soma(1/n^s,n=1,2,3...)=produto_{sobre todos os primos p} 
(1+1/p^s+1/p^{2s}+1/p^{3s}+...)

Você consegue provar a fórmula acima? (ou ao menos ver que ela tem "cara de 
ser verdadeira"?)

Aliás a soma 1+1/p^s+1/p^{2s}+1/p^{3s}+... é soma de PG, logo,

zeta(s)=produto_{sobre todos os primos p}  1/(1-p^{-s})

Considere zeta(s) como função de uma variável real definida em 
(1,infinito). Ela é contínua e lim zeta(s) para s->1 é infinito...(série 
harmonica diverge...)

A partir daí Euler deduziu que existem infinitos primos...não é difícil !

Abraço,

Bruno Leite

PS mas é claro que existem mais ligações entre zeta e teoria dos números!!!

(...)

=
Instruções para entrar na lista, sair da lista e usar a lista em
http://www.mat.puc-rio.br/~nicolau/olimp/obm-l.html
O administrador desta lista é <[EMAIL PROTECTED]>
=



Re: [obm-l] ola pessoal

2002-01-31 Thread Bruno F. C. Leite

At 18:33 31/01/02 +, you wrote:
>sou novo na lista, gostaria de ter acesso à mensagens anteriores, alguem 
>possui os arquivos?

O prof. Nicolau guarda os arquivos, acho que em 
http://www.mat.puc-rio.br/~nicolau/olimp/obm-l.html


>Gostaria de saber tb mais sobre a medalha Fields :)

Olhe http://www-groups.dcs.st-and.ac.uk/~history/Societies/FieldsMedal.html

Espero ter ajudado,

Bruno Leite
http://www.ime.usp.br/~brleite



>_
>Converse com amigos on-line, experimente o MSN Messenger: 
>http://messenger.msn.com.br
>
>=
>Instruções para entrar na lista, sair da lista e usar a lista em
>http://www.mat.puc-rio.br/~nicolau/olimp/obm-l.html
>O administrador desta lista é <[EMAIL PROTECTED]>
>=

=
Instruções para entrar na lista, sair da lista e usar a lista em
http://www.mat.puc-rio.br/~nicolau/olimp/obm-l.html
O administrador desta lista é <[EMAIL PROTECTED]>
=



[obm-l] Re: [obm-l] Livros de matemática e física

2002-02-05 Thread Bruno F. C. Leite

At 00:32 06/02/02 +, you wrote:
>Bem, eu vou contar uma pequena história de meu início "matemático" 
>desculpem se ficar muito grande, mas eu gostaria de algumas opiniões.. :)
>
>Eu sempre fui bem em matemática, mas acho que pela maneira que ela era 
>dada até o 3ano do meu colégio eu nunca tive muito interesse em estudar a 
>fundo. Até que um professor novo meu entrou, e começou a falar coisas 
>muito maneras, algo sobre Fermat ou coisa assim. Logo me interessei, e 
>procurei saber mais. Enfim, estava numa livraria e achei o livro " O 
>Último Teorema de Fermat".. resolvi comprar e devorei o livro em 2 dia, 
>adorei e a partir daí comecei a ler livros sobre Física e Matemática em 
>geral...
>
>Alguns dos títulos que li ano passado
>-Hiperespaço
>-Gigantes da Física
>-O que sabemos sobre o Universo
>-Uma breve história do Tempo
>-Visões do Futuro
>-Assim falou Einstein
>
>Todos os livros na minha opinião foram excelentes.. meu gosto pelas 
>ciências exatas aumentou muito :) Decidi fazer ITA, e esse ano entro para 
>o cursinho Poliedro, mesmo passando na Unicamp.. mas a  questão é a 
>seguinte, na minhas poucas horas de tempo livre eu gostaria de ler mais 
>livros do gênero, como biografias de grandes gênios, como Gauss, livros 
>sobre Teoremas ainda nao resolvidos, Curiosidades, Física em geral.. como 
>ainda estou começando gostaria da opinião de vocês sobre quais livros mais 
>eu poderia comprar.. embora eu more em Curitiba e aqui não tenha livrarias 
>tão boas quanto em São Paulo, esse mês eu me mudo para São José e 
>eventualmente irei para São Paulo para fazer umas comprinhas
>
>Gostaria também de saber se tem possibilidade de estudantes de cursinho 
>participar de algum tipo de olimpíada de matemática, pois apenas descobri 
>como são as provas esses meses e adorei :)

Eu li no colegial o livro "O romance das equações algébricas" (Gilberto 
garbi) e gostei. Não é um livro técnico e nem exige muitos conhecimentos ou 
paciência para ser lido. Tem várias coisas interessantes lá, uma prova do 
Teorema Fundamental da álgebra, uma prova da construtibilidade do polígono 
regular de 17 lados (com régua e compasso), a "prova" de Euler da identidade

1+1/4+1/9+1/16+1/25+...=1/6 pi^2

,etc. O livro deve custar uns R$40, eu acho.

Bruno Leite
http://www.ime.usp.br/~brleite




>Extremaente agradecido :]
>
>Rodrigo
>
>_
>O MSN Photos é o jeito mais fácil de compartilhar, editar e imprimir suas 
>fotos preferidas: http://photos.msn.com.br/support/worldwide.aspx
>
>=
>Instruções para entrar na lista, sair da lista e usar a lista em
>http://www.mat.puc-rio.br/~nicolau/olimp/obm-l.html
>O administrador desta lista é <[EMAIL PROTECTED]>
>=

=
Instruções para entrar na lista, sair da lista e usar a lista em
http://www.mat.puc-rio.br/~nicolau/olimp/obm-l.html
O administrador desta lista é <[EMAIL PROTECTED]>
=



Re: [obm-l] Re: [obm-l] Re: [obm-l] Re:_[obm-l]_ dúvida_trigonometria .

2002-02-12 Thread Bruno F. C. Leite

At 19:14 12/02/02 -0300, you wrote:
>Mas como eh q vc DEFINE concavidade sem Calculo?

Oi David,

A definição de convexidade NAO depede de calculo. definimos que f é convexa 
em um intervalo I se para todo 0<=k<=1 e shttp://www.ime.usp.br/~brleite



>-Mensagem original-
>De: Jose Paulo Carneiro <[EMAIL PROTECTED]>
>Para: [EMAIL PROTECTED] <[EMAIL PROTECTED]>
>Data: Terça-feira, 12 de Fevereiro de 2002 08:46
>Assunto: [obm-l] Re: [obm-l] Re:_[obm-l]_dúvida_trigonometria.
>
>
> >Excelente.
> >Eu ja tinha constatado que se podia usar a desigualdade das medias para
> >transferir o problema do produto de senos para a soma de senos.
> >Mas dahi por diante, como se demonstra a desigualdade de Jensen e como sabe
> >a concavidade do seno sem usar Calculo Diferencial?
> >Bom, a concavidade do seno pode-se considerar como um "dado grafico"
> >Mas valeu a elegancia da sua demonstracao.
> >JP
> >
> >
> >- Original Message -
> >From: Carlos Yuzo Shine <[EMAIL PROTECTED]>
> >To: <[EMAIL PROTECTED]>
> >Sent: Monday, February 11, 2002 10:49 PM
> >Subject: Re: [obm-l] Re:_[obm-l]_dúvida_trigonometria.
> >
> >
> >Ah, pode-se demonstrar que sen a sen b sen c <= 1/8
> >utilizando as desigualdades das médias e de Jensen.
> >
> >Só relembrando as duas desigualdades:
> >
> >A desigualdade das médias é a seguinte: dados n
> >números reais não negativos, sua média aritmética é
> >maior ou igual à média geométrica, com igualdade se, e
> >somente se, todos os n números são iguais.
> >
> >A desigualdade de Jensen é a seguinte: seja f uma
> >função com convexidade para baixo num intervalo.
> >Então, dados n números pertencentes ao intervalo, a
> >média aritmética das f's dos números é menor ou igual
> >à f da média aritmética dos números.
> >
> >A função seno tem concavidade para baixo no intervalo
> >[0;pi] e é não negativa nesse intervalo. Logo:
> >sen a sen b sen c
> > <= [(sen a + sen b + sen c)/3]^3
> > <= [sen((a+b+c)/3)]^3
> >  = [sen((pi/2)/3)]^3
> >  = 1/8
> >
> >Bom, a solução acaba dependendo um pouco de cálculo
> >para mostrar que a função sen tem concavidade para
> >baixo. Existe uma solução totalmente elementar que
> >prova que
> >  sen a sen b sen c = (1/8)*(coisas) - (mais coisas)^2
> >para a,b,c positivos, a+b+c = pi/2. Só que não lembro
> >direito a identidade.
> >
> >[]'s
> >Shine
> >
> >
> >--- Jose Paulo Carneiro <[EMAIL PROTECTED]> wrote:
> >> 1) Usando as formulas de transformacao de soma em
> >produto, voce mostra que o lado esquerdo eh igual a:
> > 4 sen 2a sen 2b sen 2c,
> >> enquanto o lado direito eh igual a:
> > 4 cos a cos b cos c.  Verifique se confere.
> >> 2) A partir dahi (e usando sen 2a = 2 sen a cos a,
> >etc.), a questao se resume a mostrar que f(a;b;c) =
> >sen a sen b sen c <= 1/8 (naturalmente, com
> >a+b+c=pi/2).
> >
> >> Agora, pergunto: posso usar Calculo Diferencial? Se
> >puder, uma aplicacao simples de multiplicadores de
> >Lagrange  mostra que o unico ponto critico de f(a;b;c)
> >com a restricao dada eh a=b=c=pi/6, onde f vale 1/8.
> >> JP
> >>
> >
> >
> >
> >__
> >Do You Yahoo!?
> >Send FREE Valentine eCards with Yahoo! Greetings!
> >http://greetings.yahoo.com
> >=
> >Instruções para entrar na lista, sair da lista e usar a lista em
> >http://www.mat.puc-rio.br/~nicolau/olimp/obm-l.html
> >O administrador desta lista é <[EMAIL PROTECTED]>
> >=
> >
> >
> >=
> >Instruções para entrar na lista, sair da lista e usar a lista em
> >http://www.mat.puc-rio.br/~nicolau/olimp/obm-l.html
> >O administrador desta lista é <[EMAIL PROTECTED]>
> >=
> >
>
>
>
>
>=
>Instruções para entrar na lista, sair da lista e usar a lista em
>http://www.mat.puc-rio.br/~nicolau/olimp/obm-l.html
>O administrador desta lista é <[EMAIL PROTECTED]>
>=

=
Instruções para entrar na lista, sair da lista e usar a lista em
http://www.mat.puc-rio.br/~nicolau/olimp/obm-l.html
O administrador desta lista é <[EMAIL PROTECTED]>
=



Re: [obm-l] Re

2002-02-21 Thread Bruno F. C. Leite

At 22:34 21/02/02 -0300, you wrote:
>Valeu pela resolução  David e demais companheiros de lista.
>
>Eu gostaria de propor mais duas:
>
>
>1)Seja f:R==>R,não identicamente nula,tal que
>
>f(x)*f(y)=(1/2)[f(x+y)+f(x-y)] e f(1)=0,para todos os números reais x e y.
>
>a)Mostre que f(0)=1,f(2)=-1,f(3)=0 e f(4)=1.
>b)Mostre que f(x+4)=f(x),para todo x real.
>c)Existe de fato tal função.
>
>É fácil verificar o item a,mas não consegui o b e o c.
>
>2)Seja p(x)=x^3+ax^2+bx+c um polinômio com coeficientes inteiros.Suponha 
>que a equação p(x)-0 tem raízes inteiras distintas.Mostre que a equação 
>p(x)-1=0 não admite nenhuma raiz inteira.

Temos que p(x)=(x-a)(x-b)(x-c). Se p(x)-1=0, então (x-a)(x-b)(x-c)=1. Como 
x,a,b,c são inteiros, ou
x-a=x-b=x-c=1 ou x-a=1 e x-b=x-c=-1, por exemplo. Mas as duas contradizem o 
fato de a,b,c serem diferentes.

>
>3)Dada uma equação do segundo grau, com coeficientes inteiros,mostre que 
>seu discriminante não pode ser igual a 23.

Há um modo mais fácil de fazer isso se vc observar os restos na divião por 
4. MODULO 4, temos
b^2=23=3, e isso nao tem solução pois nenhum quadrado deixa resto 3 na 
divisao por 4.

Não pensei na 1, mas espero ter ajudado.

Bruno Leite
http://www.ime.usp.br/~brleite


>
>
>Essa eu acho que consegui fazer.Como eu não tenho muita prática em 
>problemas de olimpíada,vou esboçar minha resolução.Quem vir alguma 
>besteira,pode comentar se quiser.
>
>Fiz y=ax²+bx+c (com a,b e c nas condições do enunciado)
>
>Observei que todo quadrado perfeito termina em 0,1,4,5,6 ou 9 e que os 
>múltiplos de 4 terminam em
>0,2,4,6 ou 8.
>
>Daí verifiquei o algarismo das unidades de delta=b²-4ac admitindo b² 
>terminando em 0,1,4,5,6 ou 9.
>
>As possibilidades de 3 ser o algarismo das unidades de delta apareceram 
>para b² terminando em 1 e b² terminando em 9.
>
>Para b² terminando em 1,temos que b termina em 1 ou 9.Daí b pertence 
>{+-1,+-9,+-11,+-19,...}.
>Esses números são da forma 4k+ -1,k inteiro.
>
>(4k+ -1)²=16k²+-8k+1
>
>Como delta =b²-4ac,fiz delta igual a 23:
>
>16k²+-8k+1-4ac=23 ==> 2(k²+-2k-ac)=11 ==> 11 é múltiplo de 2(absurdo)
>
>Para b² terminando em 5,temos b múltiplo ímpar de 5.Pondo b=5(2k+1):
>
>delta=25(4k²+4k+1)-4ac
>
>Fazendo delta=23,resulta 25k²+25k-ac=(-1/2).Mas sendo k,a e c inteiros 
>isso não pode acontecer.
>
>Delta nunca é igual a 23.
>- Original Message -
>From: David Daniel Turchick
>To: [EMAIL PROTECTED]
>Sent: Thursday, February 21, 2002 12:36 AM
>Subject: Re: [obm-l] ???
>
>Eder, eu mandei e-mail respondendo à sua dúvida prá lista, mas por algum 
>motivo ele não chegou, sei lá pq... Aí vai a minha resposta.
>
>
>
>Você conhece o Teorema de Cramer? Ele diz que um sistema linear de n 
>equações a n incógnitas tem solução única se, e somente se, o determinante 
>da matriz dos coeficientes for não-nulo.
>
>Sendo os pontos (x_1,y_1), (x_2,y_2) e (x_3,y_3), queremos encontrar a,b,c 
>reais tais que a*(x_i)^2+b*x_1+c=y_i, i=1,2,3. Acabamos então de montar um 
>sistema linear de 3 equações a 3 incógnitas, a, b e c. A matriz dos 
>coeficientes é {[(x_1)^2, x_1, 1], [(x_2)^2, x_2, 1], [(x_3)^2, x_3, 1]}, 
>cujo determinante é (x_2-x_1)*(x_3-x_1)*(x_3-x_2) (a matriz é de 
>Vandermonde, então é fácil). Isso só seria zero se tivéssemos coincidência 
>de pelo menos duas abscissas, o que você explicitou não acontecer. Logo, 
>pelo Teorema de Cramer, EXISTEM ÚNICOS a,b,c reais que satisfazem o 
>sistema. Fora isso, o a não é zero, pois se fosse, teríamos a reta bx+c 
>passando pelos três pontos (que você disse serem não-colineares). Logo, 
>existe uma única parábola passando pelos três pontos.
>
>Você pode verificar que esse argumento (até a parte do "a não é zero, 
>pois...") continua valendo para um caso mais geral: por n pontos de RxR 
>com abscissas distintas 2 a 2, passa no máximo uma função polinomial de 
>grau n-1.
>
>David
>-Mensagem original-
>De: Eder <[EMAIL PROTECTED]>
>Para: [EMAIL PROTECTED] 
><[EMAIL PROTECTED]>
>Data: Quarta-feira, 20 de Fevereiro de 2002 16:38
>Assunto: [obm-l] ???
>
>Olá,
>
>Será que alguém poderia ajudar nesta questão:
>
>"Considere três pontos no plano cartesiano,não colineares e com abcissas 
>distintas duas a duas.Qual o número de funções quadráticas que podem ser 
>encontradas de maneira que esses pontos pertençam aos seus gráficos?"
>
>Essa questão foi do vestibular de uma universidade não lá muito 
>conceituada,mas eu ainda não matei a charada...

=
Instruções para entrar na lista, sair da lista e usar a lista em
http://www.mat.puc-rio.br/~nicolau/olimp/obm-l.html
O administrador desta lista é <[EMAIL PROTECTED]>
=



Re: [obm-l] t. fundamental da algebra e 0,999...=1

2002-02-24 Thread Bruno F. C. Leite

At 02:03 24/02/02 -0300, you wrote:
>Olá colegas da lista, venho mais uma vez tentar esclarecer algumas dúvidas:
>
>1) Achei na Internet uma "demonstração elementar" do teorema fundamental 
>da algébra, q usa cálculo. O problemas é q ela cita tb coisas como anéis, 
>corpos... (o pouco q eu sei sobre isso é q têm a ver com a teoria dos 
>grupos de Galois, ou não), como já vi vários comentários sobre isso e esse 
>parece ser um assunto importantissimo quero estudar algo e gostaria de 
>referências de livros para um iniciante...Eu estava dando uma 
>olhada no arquivo da lista e encontrei uma mensagem dizendo q Gauss chegou 
>a dar 3 provas do teorema fundamental da algébra mas q todas tinham 
>considerações geoméricas e q ele queria obter uma q fosse livre dessas 
>consideraçoes, o q sao essas consideraçoes geometricas q ele utilizou? 
>alguém poderia mostrar mais ou menos o ponto de partida das demontraçoes 
>de gauss?
>
>2) 0,999...=1, essa é uma afirmação q ainda causa certa polêmica entre 
>meus colegas aqui por onde moro. Recentemente um desses colegas perguntou 
>ao seu professor de Cálculo se essa afirmação é verdadeira e ele a negou e 
>disse q se isso fosse verdade se jogava do prédio onde dá aulas. Foi a 
>maior polemica na aula. Esse colega pediu-me q renisse algo sobre tal 
>afirmaçao para q ele levasse ao tal professor. Acabei de enviar para esse 
>meu colega tudo q pude encontrar na lista sobre o assunto, (e-mails do 
>nicolau, ralph e etc.) juntamente com o endereço da lista, para ele 
>entregar ao tal professor e esse entaum tirar suas proprias conclusoes... 
>Eu nao quero retomar esse assunto aqui na lista uma vez q ele já foi muito 
>discutido, o q eu queria era pedir informação sobre q área da matematica 
>devo estudar para poder compreender melhor isso e referencias de livros

Alguém na lista (acho que o prof. Eduardo Wagner) mostrou um argumento que 
convence qualquer um. Se x e y são reais diferentes, existe ao menos um 
número real entre eles. Tente achar um número real entre 0,999... e 1...

Bruno Leite
http://www.ime.usp.br/~brleite

>
>3) qual é a equaçao do lugar geometrico dos pontos cujo produto das 
>distancias a dois outros pontos é uma constante?(como na elipse só q ao 
>inves de se somar se multiplica, se naum fui claro...). Achei uma equação 
>enorme pra se escrever aki... alguém sabe algum programa  q eu possa usar 
>para escrever equaçoes e obter graficos... de preferencia gratis (tentei 
>usar algumas simulçaoes em java na internet mas as q achei só escrevem 
>funçoes...)?
>
>[]´s hugo
>
>ps: se alguém aqui cursa ciencia da computaçao na UnB ou pode me dar 
>informaçoes sobre o curso por favor me contatem.

=
Instruções para entrar na lista, sair da lista e usar a lista em
http://www.mat.puc-rio.br/~nicolau/olimp/obm-l.html
O administrador desta lista é <[EMAIL PROTECTED]>
=



[obm-l] Mail do David (geometria)

2002-03-04 Thread Bruno F. C. Leite

Oi pessoal

O David está com problemas com o mail dele e me pediu para mandar a 
mensagem abaixo.

Bruno Leite
http://www.ime.usp.br/~brleite


-Mensagem original-
De: David Daniel Turchick <[EMAIL PROTECTED]>
Para: [EMAIL PROTECTED] <[EMAIL PROTECTED]>
Data: Quinta-feira, 28 de Fevereiro de 2002 01:37
Assunto: Re: [obm-l] GEO-prova

Olá, Josimar.

Correçãozinha: o quarto Postulado de Euclides diz que todos os ângulos 
retos são congruentes, não iguais.
Nota: a definição de ângulo reto é "aquele que é congruente a seu 
suplementar". Alguém pode pensar "Ué, pensava que era 'aquele cuja medida 
em graus é 90'!" OK, mas então vc tá usando o Postulado do Transferidor, 
que vc disse que não quer. Esse "postulado" na verdade é um teorema que 
segue da propriedade de Arquimedes, que por sua vez segue do axioma da 
continuidade (de Dedekind).
O quarto Postulado de Euclides é um teorema que só depende dos axiomas de 
incidência, estar entre e congruência, então vale, além de na Geometria 
Euclidiana, na Hiperbólica, e em Geometrias sem o axioma da continuidade. 
Vou traçar um esquema para essa prova me baseando no excelente livro 
"Euclidean and Non-Euclidean Geometries - Development and History", de 
Marvin J. Greenberg. Se está interessado nos Fundamentos da Geometria, 
realmente esse livro vale a pena!

Def.: ^ABC < ^DEF sse existe uma semi-reta EG entre as semi-retas ED e EF 
tq ^ABC < ^GEF.
Lema 1: a ordem < para ângulos é completa (ou linear), i.e., tricotômica 
(^P < ^Q ou ^P ~= ^Q ou ^P > ^Q), anti-simétrica (se ^P < ^Q, então ^Q !< 
^P) e transitiva (se ^P < ^Q e ^Q < ^R, então ^P < ^R). Também vale que se 
^P < ^Q e ^Q ~= ^R, então ^P < ^R.
Lema 2: suplementos de ângulos congruentes são congruentes.
Def.: D está no interior de ^CAB sse C e D estão no mesmo lado da reta AB e 
B e D estão no mesmo lado da reta AC. Neste caso, tb dizemos que a 
semi-reta AD está entre as semi-retas AB e AC.
Lema 3: se D está no interior de ^CAB e vale C*A*E (A está entre C e E), 
então B está no interior de ^DAE.
Agora o 4.o Post. de Euclides:
Sejam ^BAD e ^FEH retos, e ^CAD e ^GEH suplementares seus. Se ^BAD e ^FEH 
não fossem congruentes, pelo lema 2, um seria menor que o outro. Sem perda 
de generalidade, seja ^FEH < ^BAD, i.e., existe semi-reta AJ entre AB e AD 
tq ^BAJ ~= ^FEH. Pelo lema 2, ^CAJ ~= ~GEH, e como ^GEH ~= ^FEH, ^CAJ ~= 
^FEH (o 5.o axioma de congruência diz que congruência entre ângulos é uma 
relação de equivalência). Logo, ^BAJ ~= ^CAJ. Agora tome semi-reta AK entre 
as semi-retas AD e AC com ^BAJ ~= ^CAK (lema 1). Então, pelo lema 3, ^CAD < 
^CAJ ~= ^CAK < ^CAD, absurdo pelo lema 1.

Boa sorte com os lemas (o 1 é chatinho...)! Espero ter sido razoavelmente 
claro, caso contrário avise-me.
-Mensagem original-
De: Josimar <[EMAIL PROTECTED]>
Para: [EMAIL PROTECTED] <[EMAIL PROTECTED]>
Data: Terça-feira, 26 de Fevereiro de 2002 17:31
Assunto: [obm-l] GEO-prova

Olá amigos!

Adaptei o texto que segue para ser colocado num e-mail (sem anexo). 
Digitei-o há alguns anos, mas com muitos símbolos. Alguém poderia me ajudar 
como o seguinte problema?

[]s, Josimar

PROBLEMA

Apenas com os axiomas e definições abaixo, é possível provar o quarto 
postulado de Euclides?

Quarto postulado: "todos os ângulos retos são iguais entre si".

GEOMETRIA NO PLANO

I) AXIOMAS DE INCIDÊNCIA

Termos primitivos: PONTO, RETA e INCIDENTE.

Consideremos os termos "passar por", "jazer em" e suas variantes como 
sinônimos de incidentes.

AX(inc) 1 - Para todo ponto P e todo ponto Q distinto de P, existe uma 
única reta l incidente em P e Q.

AX(inc) 2 - Para toda reta l existem pelo menos dois pontos distintos 
incidentes em l.

AX(inc) 3 - Existem pelo menos três pontos distintos com a

propriedade que nenhuma reta é incidente em todos eles.

Definições

Def(inc) 1 - Dois ou mais pontos são COLINEARES quando incidem na mesma reta.

Def(inc) 2 - Duas retas são CONCORRENTES quando possuem um ponto comum, ou 
seja, quando incidem em um ponto.

Def(inc) 3 - Duas retas são PARALELAS quando não incidem em nenhum ponto 
comum, ou seja, quando não são concorrentes.

II) AXIOMAS DE ENTREMEIO (BETWEENESS)

Termo primitivo: "ESTAR ENTRE".

AX(entre) 1 - Se o ponto B está entre os pontos A e C então A, B e C são 
três pontos distintos incidentes na mesma linha reta e também B está entre 
C e A.

Introduzindo a notação A*B*C para denotar que B está entre A e C (ou, 
equivalentemente, B está entre C e A), podemos reescrever o axioma acima como:

"Se A*B*C então A, B e C são distintos e A,B,C pertencem a l e C*B*A."

AX(entre) 2 - Dados dois pontos distintos B e D, existem pontos A, C e E 
incidindo na reta l que passa por B e D e tal que A*B*D, B*C*D, B*D*E.

AX(entre) 3 - Se A, B e C são três pontos distintos incidentes em uma reta, 
então ocorre um e somente um dos casos:

i) A*B*Cii) A*C*Biii) B*A*C

Definição

Def(entre) 1 - Dizemos que dois pontos A e B estão do mesmo lado da reta l 
se o segmen

Re: [obm-l] Problemas(correção)

2002-03-08 Thread Bruno F. C. Leite

At 14:49 08/03/02 -0300, you wrote:
> 1) Encontre, em função de n , a soma dos algarismos do número
>
>   N = 9 x 99 x  x x ( 
> 10^2n  - 1) Se lê dez elevado a dois n menos 1.
>
> 2) Determine todas as funções f: Q+Q+  tais que
>
>   f(x+1) = f(x) +1  e   f(x^3) = (f(x))^3
>
>
>
> 3) Divida , justificando , o ângulo de 19º em 19 partes 
> iguais, usando somente a régua eo compasso.

Temos que construir o angulo de 1 grau, dado um angulo de 19 graus.

O angulo de 18 graus é construtivel pois o seu seno (sen 18=(raiz(5)-1)/4) 
é construtivel. Bem, com o angulo de 19 e o de 18 construímos o de 1 e acabou.

De outra maneira: (sem usar essa do sen 18) temos que 30.7-19.11=1, logo 
fazendo 7 cópias do angulo de 30 e 11 cópias do angulo de 19 (e fazendo a 
diferença) acharemos o de 1 e acabou de novo.

Bruno Leite
http://www.ime.usp.br/~brleite

=
Instruções para entrar na lista, sair da lista e usar a lista em
http://www.mat.puc-rio.br/~nicolau/olimp/obm-l.html
O administrador desta lista é <[EMAIL PROTECTED]>
=



Re: [obm-l] Provar q o produto de 3 nr consecutivos...

2002-03-08 Thread Bruno F. C. Leite

At 14:03 08/03/02 -0300, you wrote:
>Oi pessoal,
>
>   Preciso de uma ajuda:
>Prove que o produto de 3 numeros inteiros consecutivos
>é par.

na verdade, vale mais: o produto é divisível por 6. Afinal, (se n>1) 
n(n-1)(n-2)=6*binomial(n,3), e binomial(n,3) é inteiro. Dá para ver que o 
produto de k inteiros consecutivos é multiplo de k!

Bruno Leite
http://www.ime.usp.br/~brleite


>   Amplexos,
>
>   Douglas Fabiano Drumond de Carvalho
>
>UAI - ESTADO DE MINAS
>O Grande portal dos Mineiros na Internet
>http://www.uai.com.br
>=
>Instruções para entrar na lista, sair da lista e usar a lista em
>http://www.mat.puc-rio.br/~nicolau/olimp/obm-l.html
>O administrador desta lista é <[EMAIL PROTECTED]>
>=

=
Instruções para entrar na lista, sair da lista e usar a lista em
http://www.mat.puc-rio.br/~nicolau/olimp/obm-l.html
O administrador desta lista é <[EMAIL PROTECTED]>
=



Re: [obm-l] Combinatória

2002-03-21 Thread Bruno F. C. Leite

At 19:18 21/03/02 -0300, you wrote:
>Saudações a todos,
>obrigado pelas soluções ...
>
>alguém poderia ajudar?
>
>De quantos modos é possível colocar 8 damas em um tabuleiro 8x8 de modo 
>que nenhuma ataque nenhuma ?

Num curso de C que eu fiz, o professor disse que até o momento, não há 
soluções "inteligentes" - só soluções com força bruta. (Ele também disse 
que Gauss não soube fazer este problema.) Se mesmo assim vc quiser saber de 
quantos jeitos dá para fazer isso, vc pode fazer um programinha em C ou 
procurar pela internet...

Bruno Leite
http://www.ime.usp.br/~brleite

=
Instruções para entrar na lista, sair da lista e usar a lista em
http://www.mat.puc-rio.br/~nicolau/olimp/obm-l.html
O administrador desta lista é <[EMAIL PROTECTED]>
=



Re: [obm-l] SOCORROOOOOOOOOOOOOOOOOOO!!!!!!!!!!!!!!!!!

2002-03-21 Thread Bruno F. C. Leite

Oi,

Acho que isso responde só uma parte da pergunta: a da unicidade do ponto 
fixo. Temos tb a questão da existência.

Tome um ponto P qualquer do seu espaço métrico E. Seja K^n(P) a n-ésima 
iteração de K em P (K(K(K...(P)

A sequência em E (P, K(P), K^2(P),...) é de Cauchy, logo converge para um 
certo ponto F, já que E é completo. Não é dificil ver que F é ponto fixo de 
K, e é o único, pelo que já escreveram.

Bruno Leite
http://www.ime.usp.br/~brleite



At 17:31 21/03/02 -0300, you wrote:
>Olá!!!
>
>Vou responder a (3):
>
>Sejam x e y pontos fixos e K a nossa contração. Temos":
>
>K(x) = x
>
>K(y) = y
>
>Como K é contração, existe um k no intervalo (0,1) tal que:
>
>d( K(x) , K(y) ) <= kd(x,y)   (1)
>
>Mas, d( K(x), K(y) ) = d(x,y) (por serem x e y pontos fixos). Assim, a 
>nossa desigualdade (1) fica:
>
>d(x,y) <= kd(x,y)
>
>Como k pertence ao intervalo (0,1), isso implica
>
>d(x,y)=0, ou seja, x=y.
>
>Abraços,
>Claudio.
>
>At 19:32 21/03/02 +, you wrote:
>
>>Perguntas crueis:
>>
>>1)Temos um conjunto S de primos tais que se a  e b sao de S, entao ab+4 
>>tambem sera.Quantos elementos S pode ter?
>>
>>2)Qual a   prova o Teorema de Sylvester (sem usar distancias)?Qual o dual?
>>
>>3)Teorema de Banach:toda contraçao de um espaço metrico completo M)possui 
>>1 e so 1 ponto fixo.(contraçao e uma funçao K tal que 
>>d(K(x),K(y))<=k*d(x,y),para todos os x e y de M e para um certo real k 
>>fixo em ]0,1[ .d e a distancia em M.
>>
>>4)Como prova-se que o teorema da soma dos angulos do triangulo e 
>>equivalente ao postulado das paralelas?
>>
>>5)Vi uma prova do Teorema de Pitot,assim:Seja ABCD um quadrilatero 
>>circunscritivel.Prove que AB+CD=AD+BC.Lema:sendo Q a circunferencia 
>>tangente a AB,BC,CD,prove que as tangencias de Q com AB fica entre A e 
>>B.Como eu provo o dito lema?
>>
>>Valeu!!Anderson.
>>
>>
>>--
>>Converse com amigos on-line, experimente o MSN Messenger: 
>>http://messenger.msn.com.br
>>= 
>>Instruções para entrar na lista, sair da lista e usar a lista em 
>>http://www.mat.puc-rio.br/~nicolau/olimp/obm-l.html O administrador desta 
>>lista é 
>>=
>>---
>>Incoming mail is certified Virus Free.
>>Checked by AVG anti-virus system (http://www.grisoft.com).
>>Version: 6.0.333 / Virus Database: 187 - Release Date: 08/03/02
>
>
>---
>Outgoing mail is certified Virus Free.
>Checked by AVG anti-virus system (http://www.grisoft.com).
>Version: 6.0.333 / Virus Database: 187 - Release Date: 08/03/02

=
Instruções para entrar na lista, sair da lista e usar a lista em
http://www.mat.puc-rio.br/~nicolau/olimp/obm-l.html
O administrador desta lista é <[EMAIL PROTECTED]>
=



Re: [obm-l] Fatoração

2002-03-24 Thread Bruno F. C. Leite

At 15:38 24/03/02 -0800, you wrote:
>Olá Pessoal!
>
>Preciso fatorar essa expressão em dois fatores:
>x^10 + x^5 + 1

Isto é múltiplo de x^2+x+1...

Bruno Leite
http://www.ime.usp.br/~brleite



>Parece fácil, mas não consegui ainda.
>
>Obrigado,
>
>Rafael.
>
>
>=
>Rafael Werneck Cinoto
>   ICQ# 107011599
> [EMAIL PROTECTED]
>   [EMAIL PROTECTED]
>   [EMAIL PROTECTED]
>http://www.rwcinoto.hpg.com.br/
>
>__
>Do You Yahoo!?
>Yahoo! Movies - coverage of the 74th Academy Awards®
>http://movies.yahoo.com/
>=
>Instruções para entrar na lista, sair da lista e usar a lista em
>http://www.mat.puc-rio.br/~nicolau/olimp/obm-l.html
>O administrador desta lista é <[EMAIL PROTECTED]>
>=

=
Instruções para entrar na lista, sair da lista e usar a lista em
http://www.mat.puc-rio.br/~nicolau/olimp/obm-l.html
O administrador desta lista é <[EMAIL PROTECTED]>
=



Re: [obm-l] Re: [obm-l] sum(1/k^2)

2002-04-04 Thread Bruno F. C. Leite

Há um artigo na página http://www.maths.ex.ac.uk/~rjc/rjc.html, na seção 
"Miscellaneous articles and surveys": "Evaluating zeta(2)", que demostra 
isso de 14 maneiras diferentes!

O link direto é http://www.maths.ex.ac.uk/~rjc/etc/zeta2.dvi
ou
http://www.maths.ex.ac.uk/~rjc/etc/zeta2.ps
ou
http://www.maths.ex.ac.uk/~rjc/etc/zeta2.pdf

Espero ter ajudado.

Bruno Leite
http://www.ime.usp.br/~brleite



At 12:25 04/04/02 -0300, you wrote:

>árdua tarefa..
>
>-- Mensagem original --
>
> >O Paulo Santa Rita já respondeu isso. Procure nos arquivos.
> >
> >[EMAIL PROTECTED] wrote:
> >
> >>sabemos que sum(1/k^2), k=1 até infinito = pi^2/6
> >>
> >>alguém sabe me dizer pq ???
> >>
> >>agradeço desde já
> >>
> >>Gabriel Haeser
> >>www.gabas.cjb.net
> >>
> >>
> >>
> >>"Mathematicus nascitur, non fit"
> >>Matemáticos não são feitos, eles nascem
> >>
> >>
> >>--
> >>Use o melhor sistema de busca da Internet
> >>Radar UOL - http://www.radaruol.com.br
> >>
> >>
> >>
> >>=
> >>Instruções para entrar na lista, sair da lista e usar a lista em
> >>http://www.mat.puc-rio.br/~nicolau/olimp/obm-l.html
> >>O administrador desta lista é <[EMAIL PROTECTED]>
> >>=
> >>
> >>
> >
> >
> >=
> >Instruções para entrar na lista, sair da lista e usar a lista em
> >http://www.mat.puc-rio.br/~nicolau/olimp/obm-l.html
> >O administrador desta lista é <[EMAIL PROTECTED]>
> >=
> >
>
>"Mathematicus nascitur, non fit"
>Matemáticos não são feitos, eles nascem
>
>
>--
>Use o melhor sistema de busca da Internet
>Radar UOL - http://www.radaruol.com.br
>
>
>
>=
>Instruções para entrar na lista, sair da lista e usar a lista em
>http://www.mat.puc-rio.br/~nicolau/olimp/obm-l.html
>O administrador desta lista é <[EMAIL PROTECTED]>
>=

=
Instruções para entrar na lista, sair da lista e usar a lista em
http://www.mat.puc-rio.br/~nicolau/olimp/obm-l.html
O administrador desta lista é <[EMAIL PROTECTED]>
=



Re: [obm-l] Questão:

2002-04-09 Thread Bruno F. C. Leite

At 23:39 09/04/02 -0300, you wrote:
>Alguém tem uma solução para o seguinte problema?
>Mostre que a sucessor do produto de quatro inteiros positivos consecutivos 
>é sempre um quadrado perfeito.

veja só:

n(n+1)(n+2)(n+3)+1= n(n+3) (n+1)(n+2)+1=(n^2+3n)(n^2+3n+2)+1=(n^2+3n+1)^2

(usei que x(x+2)+1=(x+1)^2)

Bruno Leite
http://www.ime.usp.br/~brleite





>_
>Associe-se ao maior serviço de e-mail do mundo através do MSN Hotmail. 
>http://www.hotmail.com/br
>
>=
>Instruções para entrar na lista, sair da lista e usar a lista em
>http://www.mat.puc-rio.br/~nicolau/olimp/obm-l.html
>O administrador desta lista é <[EMAIL PROTECTED]>
>=

=
Instruções para entrar na lista, sair da lista e usar a lista em
http://www.mat.puc-rio.br/~nicolau/olimp/obm-l.html
O administrador desta lista é <[EMAIL PROTECTED]>
=



Re: [obm-l] continuidade

2002-04-11 Thread Bruno F. C. Leite

At 17:28 11/04/02 -0300, you wrote:
>Ola pessoal:
>Este exercicio eh para quem jah viu continuidade.
>"Um ciclista fez um percurso de 6 milhas em 30 minutos.
>Prove
>que, algum trecho do percurso, medindo uma milha, foi percorrido
>pelo ciclista em exatamente 5 minutos."

Vamos definir
f(x)= tempo gasto para ir de x a x+1. (x em milhas)
ou se (55 entre 0 e 5. Mas ambas contradizem o fato de ele ter feito o 
percurso de 6 milhas em 30 minutos.

Outra solução pode ser conseguida se você definir g(t)=posição no tempo 
(t+5)-posição no tempo t.
Queremos mostrar que g(t)=1 para algum t. O resto é igual...

Está tudo certo?

Bruno Leite
http://www.ime.usp.br/~brleite

>Abracos a todos,
>Luiz Alberto
>
>=
>Instruções para entrar na lista, sair da lista e usar a lista em
>http://www.mat.puc-rio.br/~nicolau/olimp/obm-l.html
>O administrador desta lista é <[EMAIL PROTECTED]>
>=

=
Instruções para entrar na lista, sair da lista e usar a lista em
http://www.mat.puc-rio.br/~nicolau/olimp/obm-l.html
O administrador desta lista é <[EMAIL PROTECTED]>
=



[obm-l] Re:

2002-04-12 Thread Bruno F. C. Leite

At 14:46 12/04/02 -0300, you wrote:
>Oi todos!!! Tenho perguntas crueis e matadoras na mao(ou no mail...)
>01)Como posso assinar a CRUX Mathematicorum?
>02)Se S e um conjunto de primos tal que se p,q sao de S(p=q ou p>q) entao
>pq+4 tambem esta em S,quantros elementos S tem?Generalize o 4.
>04)Se x+y+z=1 para x,y,z reais >0,prove que  27(xxy+yyz+zzx)<=4 e 
>determine
>a igualdade.
>05)Como se prova o teorema dos 4 Quadrados(qualquer natural e a soma de
>4 quadrados perfeitos)e dos 5 Cubos?

Que teorema dos 5 cubos é esse?

Bruno Leite
http://www.ime.usp.br/~brleite



>06)Como faço para abrir os artigos ps e zip da Semana Olimpica?
>
>
>
>
>
>_
>eMTV: receba a mordomia eletrônica!
>http://mtv.uol.com.br/emtv
>
>
>
>=
>Instruções para entrar na lista, sair da lista e usar a lista em
>http://www.mat.puc-rio.br/~nicolau/olimp/obm-l.html
>O administrador desta lista é <[EMAIL PROTECTED]>
>=

=
Instruções para entrar na lista, sair da lista e usar a lista em
http://www.mat.puc-rio.br/~nicolau/olimp/obm-l.html
O administrador desta lista é <[EMAIL PROTECTED]>
=



Re: [obm-l] Número neperiano

2002-04-19 Thread Bruno F. C. Leite

At 02:18 19/04/02 -0300, you wrote:

> Como se explica o que é número neperiano p/um aluno do 3º ano do
>Ensino Médio (ou seja, um vrestibulando)? Vale lembrar que o sujeito NÃO
>está familiarizado com log e NUNCA viu exp... Obviamente, dizer q é a
>base de log resultante da integral de 1/x tb não vale :0)
>
>[]'s

Você está pedindo uma definição de E que não use log ou exp? Se sim, 
pode-se usar
E=soma(1/k!),k=0,1,2..

ou

E=limite(1+1/n)^n

Tudo bem que as duas envolvem limites, mas é mais ou menos intuitivo para 
um vestibulando (ele já viu a soma dasérie geométrica). Dá para mostrar 
elementarmente que a série de 1/K! converge, pq k!>2^k para k>=4.

Bruno Leite
http://www.ime.usp.br/~brleite



>Alexandre Tessarollo
>
>=
>Instruções para entrar na lista, sair da lista e usar a lista em
>http://www.mat.puc-rio.br/~nicolau/olimp/obm-l.html
>O administrador desta lista é <[EMAIL PROTECTED]>
>=

=
Instruções para entrar na lista, sair da lista e usar a lista em
http://www.mat.puc-rio.br/~nicolau/olimp/obm-l.html
O administrador desta lista é <[EMAIL PROTECTED]>
=



Re: [obm-l] Re:

2002-04-19 Thread Bruno F. C. Leite

Desenhando com cuidado, vemos que existem 3 soluções. Da outra vez que este 
problema passou na lista, acharam a raiz negativa por métodos numéricos.

Bruno Leite
http://www.ime.usp.br/~brleite


  At 13:47 19/04/02 +, you wrote:

>  Já vi esta questão antes e são 3 soluções reais; 2 e 4 são fáceis de 
> serem vistas, mas existe uma terceira...alguém consegue achar??
>  []´s
>Fê
>
>
>
>
>> Essa eu já vi diversas similares mas até hoje não aprendi a fazer esse
>>tipo de questão... Mas, se for te ajudar, x=2 é uma soluçào "óbvia" do
>>equação. Olhando pelo gráfico de x^2 e 2^x (um tanto similar a da exp(x)),
>>vemos que eles se cortam em apenas dois pontos. Resta agora achar o outro.
>>
>>Parêntesis
>> Momento infame e infeliz daquele professor q não sabe responder: 
>> "Pronto,
>>já resolvi metade do prob com x=2 e indiquei o caminho para a segunda raiz.
>>Agora o resto é com vc"
>>Fim do(s) parêntesis
>>
>>[]'s
>>
>>Alexandre Tessarollo
>>
>>=
>>Instruções para entrar na lista, sair da lista e usar a lista em
>>http://www.mat.puc-rio.br/~nicolau/olimp/obm-l.html
>>O administrador desta lista é <[EMAIL PROTECTED]>
>>=
>
>
>_
>O MSN Photos é o modo mais fácil de compartilhar e imprimir suas fotos: 
>http://photos.msn.com/support/worldwide.aspx
>
>=
>Instruções para entrar na lista, sair da lista e usar a lista em
>http://www.mat.puc-rio.br/~nicolau/olimp/obm-l.html
>O administrador desta lista é <[EMAIL PROTECTED]>
>=

=
Instruções para entrar na lista, sair da lista e usar a lista em
http://www.mat.puc-rio.br/~nicolau/olimp/obm-l.html
O administrador desta lista é <[EMAIL PROTECTED]>
=



Re: [obm-l] funça Zeta de Riemman.

2002-04-19 Thread Bruno F. C. Leite

Oi Gabriel,

Eu escrevi umas coisas sobre este assunto um tempinho atrás. Você pode 
procurar por emails com o titulo "Teorema de fermat" nos arquivos da lista, 
no fim de janeiro de 2002.

Mas o principal é que vale o seguinte, para todo s complexo:
$\zeta(1-s)=2(2\pi)^{-s} \gamma(s)\cos(s\pi /2)\zeta(s)$, e dos zeros do 
cosseno você deduz os zeros "triviais" de zeta. (na verdade é preciso saber 
também que zeta tem um único pólo em s=1 e é holomorfa no resto do plano, 
pq senão um zero poderia ser "estragado" por um pólo, etc)

As mensagens que eu citei têm mais detalhes. Inclusive, na sequencia, e 
também com o título "Teorema de Fermat", o Gugu observou que a série dos 
inversos dos primos diverge, que era uma coisa que alguém na lista 
perguntou esses dias.

Bruno Leite
http://www.ime.usp.br/~brleite

At 14:40 19/04/02 -0300, you wrote:
>zeta(s)=sum(1/k^s),k=1 até infinito
>
>sei que esse assunto já foi amplamente discutido aqui na lista porém não
>consegui encontrar os arquivos .. mas será que alguém poderia me ajudar
>a encontrar os zeros triviais ??
>
>obrigado !
>
>"Mathematicus nascitur, non fit"
>Matemáticos não são feitos, eles nascem
>---
>Gabriel Haeser
>www.gabas.cjb.net
>
>
>--
>Use o melhor sistema de busca da Internet
>Radar UOL - http://www.radaruol.com.br
>
>
>
>=
>Instruções para entrar na lista, sair da lista e usar a lista em
>http://www.mat.puc-rio.br/~nicolau/olimp/obm-l.html
>O administrador desta lista é <[EMAIL PROTECTED]>
>=

=
Instruções para entrar na lista, sair da lista e usar a lista em
http://www.mat.puc-rio.br/~nicolau/olimp/obm-l.html
O administrador desta lista é <[EMAIL PROTECTED]>
=



Re: [obm-l] Estudos_sobre_Equações

2002-04-23 Thread Bruno F. C. Leite

At 15:05 23/04/02 -0700, you wrote:
>Quanto à primeira questão eu fiz o seguinte:
>x.(x + 1).(x² + x + 1) = 42
>x.(x + 1).(x² + x + 1) - 42 = 0
>(x² + x).(x² + x + 1) - 42 = 0

Acho que também poderia ser feito o seguinte: chame y=x^2+x. Aí, 
y(y+1)=42,  achamos y, depois achamos x.

Bruno Leite
http://www.ime.usp.br/~brleite


>x^4 + 2x³ + 2x² + x - 42 = 0
>
>Que por Briot-Ruffini podemos pegar os divisores de 42
>para testar. Veja que vale para x = 2. Simplificando:
>(x - 2).(x³ + 4x² + 10x + 21) = 0
>
>Novamente por Briot-Ruffini podemos pegar os divisores
>de 21 para testar no segundo vator. Veja que vale para
>x = -3. Simplificando:
>(x - 2).(x³ + 4x² + 10x + 21) = 0
>(x - 2).(x + 3).(x² + x + 7) = 0
>
>Como (x² + x + 7) = 0 não admite raiz real só há duas
>raízes: -3 e 2.
>
>Só não sei se é dada a regra de Briot-Ruffini no
>pimeiro grau...
>
>Rafael.
>
>
>--- Jose Francisco Guimaraes Costa
><[EMAIL PROTECTED]> wrote:
> > prob 1: 2 raizes reais: -3,18416 e 2,03165
> >
> > prob 2: nenhuma raiz real
> >
> > JF
> >
> > PS para Morgado, o Ainda Vivo:
> >
> > V ensina seus alunos do primeiro grau como resolver
> > esses problemas? Nem
> > Morgado, o Já Morto, nem Cecil Thiré me ensinaram!
> >
> >
> > -Mensagem Original-
> > De: <[EMAIL PROTECTED]>
> > Para: <[EMAIL PROTECTED]>
> > Enviada em: Segunda-feira, 22 de Abril de 2002 22:43
> > Assunto: [obm-l] Estudos sobre Equações
> >
> >
> > > Olá amigos..
> > > Ai vão alguns problemas interessantes de
> > equações..
> > > Se puderem me dar uma luz...
> > >
> > > 1-
> > > O número de raízes reais da equação
> > > x.(x + 1).(x² + x + 1) = 42
> > >
> > > 2-
> > > O número de raízes reais da equação
> > > 3x^4 - 2x³ + 4x² - 4x + 12 = 0
> > >
> > (...)
> > >
> > > São todos exercícios muito bons , com conhecimento
> > a nível de 1° grau ,
> > > eu não consegui enxergar uma solução válida.
> > > Obrigado..
> > > Rick Barbosa
> >
> >
> >
>=
> > Instruções para entrar na lista, sair da lista e
> > usar a lista em
> > http://www.mat.puc-rio.br/~nicolau/olimp/obm-l.html
> > O administrador desta lista é
> > <[EMAIL PROTECTED]>
> >
>=
>
>
>=
>Rafael Werneck Cinoto
>ICQ# 107011599
>  [EMAIL PROTECTED]
>[EMAIL PROTECTED]
>[EMAIL PROTECTED]
>http://www.rwcinoto.hpg.com.br/
>
>__
>Do You Yahoo!?
>Yahoo! Games - play chess, backgammon, pool and more
>http://games.yahoo.com/
>=
>Instruções para entrar na lista, sair da lista e usar a lista em
>http://www.mat.puc-rio.br/~nicolau/olimp/obm-l.html
>O administrador desta lista é <[EMAIL PROTECTED]>
>=

=
Instruções para entrar na lista, sair da lista e usar a lista em
http://www.mat.puc-rio.br/~nicolau/olimp/obm-l.html
O administrador desta lista é <[EMAIL PROTECTED]>
=



Re: [obm-l] Algumas da Iberoamericana.SEGUNDO PROBLEMA PARA A LISTA

2002-04-30 Thread Bruno F. C. Leite

At 17:07 30/04/02 -0300, you wrote:
>Ah.turma,to com a prova da Iberoamericana aquoi na mao,e tenho problemas
>serios neles.Ai vai!!!
>1.Temos 98 pontos sobre uma circunferencia.Maria e Jose fazem um jogo 
>assim:cada
>um deles traça uma corda ligando dois dos pontos dados que nao tenham sido
>ligados entre si antes.O jogo acaba quandoos 98 pontos forem usados como
>extremos de segmentos pelo menos 1 vez.Quem fizer o ultimo traço ganha.Defina
>uma estrategia vencedora se ela existir.
>2.B e um inteiro maior que 10 que so tem 1,3,7,9 como digitos decimais.Prove
>que B tem um fator primo maior que 11.(o que consegui chegar foi nisso:se
>p>11 acarretasse B/p nao inteiro,entao os unicos fatores primos de B sao
>3 e 7.Oras,5 nao pode,ja que 5*K==0 ou 5(mod 10).E 2 tambem nao,ja que 
>2*k==0,2,4,6,8
>(mod 10).

Já resolvi este problema uma vez (faz tempo) e, pelo que me lembro, vc 
supoe que os unicos primos que dividem B são 3 e 7, logo B=3^a 7^b. Aí 
(tomara que seja isso mesmo) vc prova que todo numero
da forma 3^a 7^b e maior que 10 tem o digito das dezenas par!

Bruno Leite
http://www.ime.usp.br/~brleite


>Problema:seja ABC um triangulo de incentro I.Nele se desenha o circuncirculo
>de BIC.Ache o ponto I sabendo desse circuncirculo
>e do ABC usando uma regua lisa.
>
>
>
>--
>Use o melhor sistema de busca da Internet
>Radar UOL - http://www.radaruol.com.br
>
>
>
>=
>Instruções para entrar na lista, sair da lista e usar a lista em
>http://www.mat.puc-rio.br/~nicolau/olimp/obm-l.html
>O administrador desta lista é <[EMAIL PROTECTED]>
>=

=
Instruções para entrar na lista, sair da lista e usar a lista em
http://www.mat.puc-rio.br/~nicolau/olimp/obm-l.html
O administrador desta lista é <[EMAIL PROTECTED]>
=



Re: [obm-l] Inducao

2002-05-10 Thread Bruno F. C. Leite

At 23:26 09/05/02 -0300, you wrote:
>Oi,
> Estou com problemas nos conceitos do metodo de prova da inducao
>matematica, alguem poderia ajduar? Vejam os exemplos abaixo e por favor
>tentem me explicar o q esta errado ... ah, os problemas foram tirados do
>livro do knuth...

Que livro do Knuth?


>1) Let "a" be any positive number. For all positive integers "n" we have
>a^(n-1) = 1.
>Proof: If n = 1, a^(n-1) = 1. And by induction, assuming that the theorem is
>true for 1, 2, 3 ..., n, we have:
>a^[(n+1) - 1] = a^n = a^(n-1) * a^(n-1) / a^(n-2) = 1*1/1 = 1
>
>Onde esta o erro da prova de acordo com a definicao de inducao? Parece claro
>q a hipotese a^(n-1) nao e valida para todo n, mas pela definicao de inducao
>e necessario tambem provar para n=2? Ha tb o problema do termo a^(n-2) nao
>estar definido para n=1, mas se ele estivesse definido como a^(n-2) = 1 a
>prova estaria correta?

O problema parece ser o seguinte: a^(n-2) não é inteiro positivo se n =1 
logo o começo da indução está errado. (vc está usando uma hipótese que NAO 
é a hipotese de indução)


>2) The following proof by induction seems correct, but for some reason the
>equation for n = 6 gives
>1/2 + 1/6  + 1/12 / + 1/20 + 1/30 = 5/6 on the left-hand sid, and 3/2 - 1/6
>= 4/3 on the right-hand side. Find a mistake:
>
>Theorem:
>1/1*2 + 1/2*3 + 1/3*4 + ... + 1/(n-1)*n = 3/2 - 1/n
>
>Proof:  We use induction on n. For n = 1, 3/2 - 1/n = 1/1*2 and, assuming
>the theorem is true for n,
>
>1/1*2 + 1/2*3 + 1/3*4 + ... + 1/(n-1)*n  + 1/n*(n+1)
>
>= 3/2 - 1/n + 1/n*(n+1) = 3/2 -1/n + [1/n - 1/(n+1)] = 3/2 - 1/(n+1)
>
>Nesse eu so vi o problema do termo (n-1) nao estar definido para todo "n"
>... sera so este o problema?

Sim, mas este já é um problema grave:
1/1*2 + 1/2*3 + 1/3*4 + ... + 1/(n-1)*n = 3/2 - 1/n
NAO VALE para n=1.

Por outro lado, na indução vc usa que vale para 1 para provar que vale para 
2 para provar que vale para 3...se vc partir de uma bobagem, pode chegar em 
bobagem.

Por exemplo. é fácil provar por indução que para todo n inteiro positivo, 
n+10=n, se assumirmos que vale para n=1. Afinal, n+1+10=n+1 se e só se n+10=n.

Isso é diferente de começarmos a indução com um número que não seja 1. Por 
exemplo: "prove que se n>=4, n!>2^n" Aqui, se n=1,2 ou 3, o que queremos 
provar é falso, mas isso não atrapalha pq nem usaremos estes casos para 
completar a indução.

Bruno Leite
http://www.ime.usp.br/~brleite



>Obrigado,
>Anderson
>
>
>
>
>
>
>=
>Instruções para entrar na lista, sair da lista e usar a lista em
>http://www.mat.puc-rio.br/~nicolau/olimp/obm-l.html
>O administrador desta lista é <[EMAIL PROTECTED]>
>=

=
Instruções para entrar na lista, sair da lista e usar a lista em
http://www.mat.puc-rio.br/~nicolau/olimp/obm-l.html
O administrador desta lista é <[EMAIL PROTECTED]>
=



Re: [obm-l] Lista para treino....

2002-05-13 Thread Bruno F. C. Leite

At 19:26 13/05/02 -0400, you wrote:
>1)Em uma dessas listas pra treino para olimpíadas, o sujeito pede para 
>calcularmos a soma de todos os divisores positivos de n? Existe alguma 
>fórmula para isso?

existe, em termos da fatoração de n em primos. Fica fácil se vc provar que 
se mdc(a,b)=1, então soma_divisores(a) 
soma_divisores(b)=soma_divisores(ab). (e aí só falta saber 
soma_divisores(p^n), onde p é primo- mas isso é fácil mesmo)

>2)Sendo N o número de divisores positivos de n, determine, em função de n 
>e N o produto de todos os divisores de n.

Se d divide n, n/d divide n. Agrupe os divisores dessa forma...(não esqueça 
o caso em que n é quadrado)

>3)Mostre que qualquer P.A, não constante, de inteiros possui uma 
>infinidade de valores compostos.

Suponha que só possua finitos valores compostos. Então, a partir de um 
ponto, todos os valores da PA são primos. Seja a+kb a sua PA, com 
k=0,1,2 Se k>=k_0, então a+bk é primo. Temos que a deve ser ímpar e b 
par. Tome k=ak_0>=k_0. Então a+bak_0 é primo, logo a=1. Agora tomando
k=2b^(n-1)+b^(2n-1), a+bk=1+2b^n+b^(2n)=(b^n+1)^2 é primo se n for 
suficientemente grande, o que é absurdo.

Está confuso (e deve ter solução mais simples) mas acho que está certo.

Bruno Leite
http://www.ime.usp.br/~brleite


>Agradeço de antemão a quem resolver.
>   Crom

=
Instruções para entrar na lista, sair da lista e usar a lista em
http://www.mat.puc-rio.br/~nicolau/olimp/obm-l.html
O administrador desta lista é <[EMAIL PROTECTED]>
=



Re: [obm-l] Apelo: Mais da Iberoamericana

2002-05-16 Thread Bruno F. C. Leite

At 14:21 16/05/02 -0300, you wrote:
>Meu,que coisa e essa?Ate agora ninguem me respondeu

Ninguém é pago para isso.

Bruno


>-- Mensagem original --
>
> >Alo turma!!Tenho mais perguntas a fazer(da Iberoamericana):
> >1)Ache todos os naturais n de 3,2 ou 1 digito tal que o quadrado de n seja
> >o cubo da soma dos digitos.
> >2)Encontre o menor n tal que se pegarmos n dos 999 primeiros inteiros 
> positivos
> >sempre se acham 4 numeros diferentes a,b,c,d com a+2*b+3*c-4*d=0.
> >3)L>0 e tal que -L^2+1998*L+1=0.Seja a recorrencia a(0)=1 e a(n+1)=parte
> >inteira de L*a(n)=[L*a(n)].Calcule a(1998)mod  1998(x mod y e o resto de
> >x por y).
> >4)na mesa da banca de lideres da OIM estao lideres de P paises de modo
>que
> >se dois lideres quaisquer sao de mesmo pais entao seus vizinhos direitos
> >nao sao.Quantos lideres ha no maximo?
> >5)Sabe-se que num conjunto de primos se p e q sao elementos(iguais ou 
> nao)entao
> >p*q+r,em que r e constante.Quantos elementos tem S com n=4?Generalize o
> >r.
> >
> >TRANSIRE SVVM PECTVS MVNDOQUE POTIRE
> >CONGREGATI EX TOTO ORBE MATHEMATICI OB SCRIPTA INSIGNIA TRIBVERE
> >Medalha Fields(John Charles Fields)
> >
> >
> >--
> >Use o melhor sistema de busca da Internet
> >Radar UOL - http://www.radaruol.com.br
> >
> >
> >
> >=
> >Instruções para entrar na lista, sair da lista e usar a lista em
> >http://www.mat.puc-rio.br/~nicolau/olimp/obm-l.html
> >O administrador desta lista é <[EMAIL PROTECTED]>
> >=
> >
>
>TRANSIRE SVVM PECTVS MVNDOQUE POTIRE
>CONGREGATI EX TOTO ORBE MATHEMATICI OB SCRIPTA INSIGNIA TRIBVERE
>Medalha Fields(John Charles Fields)
>
>
>--
>Use o melhor sistema de busca da Internet
>Radar UOL - http://www.radaruol.com.br
>
>
>
>=
>Instruções para entrar na lista, sair da lista e usar a lista em
>http://www.mat.puc-rio.br/~nicolau/olimp/obm-l.html
>O administrador desta lista é <[EMAIL PROTECTED]>
>=

=
Instruções para entrar na lista, sair da lista e usar a lista em
http://www.mat.puc-rio.br/~nicolau/olimp/obm-l.html
O administrador desta lista é <[EMAIL PROTECTED]>
=



Re: [obm-l] Livros de Inducao / Analitica

2002-06-07 Thread Bruno F. C. Leite

At 16:22 06/06/02 -0300, you wrote:
>Olá amigos.
>
>Alguem poderia me indicar um livro/site que explique o método de Inducao
>Matematica? Quero um livro que nao seja o "Matematica Elementar". Estou
>com duvidas principalmente (mas nao somente) em provar que inequacoes sao
>verdadeiras.

Há um artigo, acho que na eureka1, sobre isso. além disso, no site 
http://www.maths.uwa.edu.au/~gregg/Academy/
tem umas coisas sobre indução


>Abusando, poderiam me falar um livro de Geometria Analitica (com
>tratamento de vetores) diferente do do Elon?

No IME-USP usam o "Geometria Analítica - um tratamento vetorial", de Boulos 
e Camargo

Bruno Leite
http://www.ime.usp.br/~brleite



>=
>[]s
>Ricardo Miranda
>Matematica - UFV
>[EMAIL PROTECTED]
>http://rm2.hpg.ig.com.br/
>
>___
>Copa 2002
>Yahoo! - Patrocinador oficial da Copa do Mundo da FIFA 2002
>http://br.sports.yahoo.com/fifaworldcup/
>=
>Instruções para entrar na lista, sair da lista e usar a lista em
>http://www.mat.puc-rio.br/~nicolau/olimp/obm-l.html
>O administrador desta lista é <[EMAIL PROTECTED]>
>=

=
Instruções para entrar na lista, sair da lista e usar a lista em
http://www.mat.puc-rio.br/~nicolau/olimp/obm-l.html
O administrador desta lista é <[EMAIL PROTECTED]>
=



Re: [obm-l] Axiomas de Peano

2002-06-18 Thread Bruno F. C. Leite

At 15:29 18/06/02 -0300, you wrote:
>Na Eureka 3, p. 26,  há um artigo de Elon Lages Lima chamado "O Princípio da
>Indução", onde o autor afirma que o conjunto N dos números naturais é
>caracterizado pelas seguintes propriedades:
>
>A) Existe função s: N -> N, que associa a cada n pertencente a N um elemento
>s(n) pertecente a N, chamado o sucessor de n.
>
>B) A função s: N-> N é injetiva.
>
>C) Existe um único elemento 1 no conjunto N, tal que 1 != s(n) para todo n
>pertencente a N.
>
>D) Se um subconjunto X contido em N é tal que 1 pertence a N e s(X) está
>contido em X.

Não me lembro do artigo, mas isto está certo mesmo?
  Acho que o certo é "se um subconjunto X contido em N é tal que 1 pertence 
a N e se n está em X implica que s(n) também está, então X=N" (princípio de 
indução)

Com isso o conjunto que você falou (V) não satisfaz a última condição.

Bruno Leite
http://www.ime.usp.br/~brleite



>As afirmações A, B, C e D são os axiomas de Peano.
>
>Agora vem a minha dúvida. Imagine o conjunto de números:
>V = {0, 1, 2, 3, ...} U {a}, onde o elemento 'a' não pertence a {0, 1, 2, 3,
>...}
>e a função injetiva s: V -> V onde:
>s(x) = a, se x=a; senão s(x) = x+1
>
>Temos, então, o conjunto V e a função s que satisfazem os axiomas de Peano.
>Dessa forma, podemos dizer que V é o conjunto dos número naturais, mas não
>é!
>Qual o problema aí???
>
>Alguém pode esclarecer a minha dúvida?
>
>Obrigado
>
>Vinicius Fortuna
>
>
>
>
>=
>Instruções para entrar na lista, sair da lista e usar a lista em
>http://www.mat.puc-rio.br/~nicolau/olimp/obm-l.html
>O administrador desta lista é <[EMAIL PROTECTED]>
>=

=
Instruções para entrar na lista, sair da lista e usar a lista em
http://www.mat.puc-rio.br/~nicolau/olimp/obm-l.html
O administrador desta lista é <[EMAIL PROTECTED]>
=